Win up to 100% Scholarship

Register Now

Geography

 

2022

 

Question 1

In the northern hemisphere, the longest day of the year normally occurs in the:

(a) First half of the month of June

(b) Second half of the month of June

(c) First half of the month of July

(d) Second half of the month of July

Ans: b

Sub-Theme: Motion of Earth

Summer Solstice (21st June):

  • The Northern Hemisphere is tilted towards the sun and the rays of the sun fall directly on the Tropic of Cancer → areas receive more heat.
  • The longest day and the shortest night at these places occur on 21st June. Thus, the second half of the month of June in the northern hemisphere has the longest day of the year.
  • Areas near the poles receive less heat as the rays of the sun are slanting.
  • The North Pole is inclined towards the sun and the places beyond the Arctic Circle experience continuous daylight for about six months.
  • Since a large portion of the Northern Hemisphere is getting light from the sun, it is summer in the regions north of the equator.
  • Southern Hemisphere: All these conditions are reversed (winter season- longer nights)
NOTE: This question is a repetition from 2019 only the sentences of both the question and options are twisted to confuse the candidate. Also UPSC has asked questions from similar themes in 2013, please refer to the PYQs for detailed explanation.

 

Question 2

Consider the following statements:

1. High clouds primarily reflect solar radiation and cool the surface of the Earth.

2. Low clouds have a high absorption of infrared radiation emanating from the Earth’s surface and thus cause a warming effect.

Which of the statements given above is/are correct?

(a) 1 only

(b) 2 only

(c) Both 1 and 2

(d) Neither 1 nor 2

Ans: d

Sub-Theme: Clouds

Statement 1 is incorrect: Low, thick clouds primarily reflect solar radiation and cool the surface of the Earth.

Statement 2 is incorrect: High, thin clouds primarily transmit incoming solar radiation. At the same time, they trap some of the outgoing infrared radiation emitted by the Earth and radiate it back downward, thereby warming the surface of the Earth.

  • Whether a given cloud will heat or cool the surface depends on several factors, including the cloud’s altitude, its size, and the make-up of the particles that form the cloud.
  • The components of the Earth system that are important to the radiation budget are the planet’s surface, atmosphere, and clouds.

 

Question 3

Consider the following States:

  1. Andhra Pradesh

  2. Kerala

  3. Himachal Pradesh

  4. Tripura

How many of the above are generally known as tea- producing States?

(a) Only one State

(b) Only two States

(c) Only three States

(d) All four States

Ans: c

Sub-Theme: Tea production

  • Black soil is ideal for growing cotton and thus it is also known as black cotton soil.
  • Formed as a result of the basaltic rocks’ weathering, which first appeared during Cretaceous fissure eruption.
  • This type of lava flow-derived soil is typical of the northwest Deccan plateau’s Deccan trap (Basalt) region.
  • Basalt contains titanium-ferromagnetic chemicals, which give basalt its characteristic black colour.

 

Tea Climatic conditions:

  • Climate: Tropical and Subtropical climate, Warm and moist frost free climate throughout the year with rainfall-1500
  • Soil: Tea plants require well-drained, fertile soil that is rich in organic matter, The pH of the soil should be slightly acidic, ranging between 4.5 and 5.5,The soil should be able to retain moisture while also allowing excess water to drain away, Tea plants prefer soils with good structure and high porosity to allow oxygen to reach the roots.
  • Temperature: Less than 15 degree Celsius.
  • As per the Tea Board under the Ministry of Commerce and Industry, and Indian Tea Association, Assam, West Bengal, Tamil Nadu, Himachal Pradesh, Tripura, Kerala and Karnataka are the states known as tea- producing States.

Picture3 2

 

Question 4

With reference to India, consider the following statements:

  1. Monazite is a source of rare earths.

  2. Monazite contains thorium.

  3. Monazite occurs naturally in the entire Indian coastal sands in India.

  4. In India, government bodies only can process or export monazite.

Which of the statements given above are correct?

(a) 1, 2 and 3 only

(b) 1, 2 and 4 only

(c) 3 and 4 only

(d) 1, 2, 3 and 4

Ans: b

Sub-Theme: Rare Earth Metal

Statement 1 and 2 are correct: Rare earth metals are a group of 17 elements. They are lustrous silvery-white soft heavy metals. China accounts for 90% of the world’s rare earth production. In India, monazite is the principal source of rare earths and thorium.

Statement 3 is incorrect: Monazite, though found in most coastal areas of India, is unlikely to be found along the entire coast. The main mines are found along the coasts of southern India in Kerala, Tamil Nadu and in Orissa.

Statement 4 is correct: As per Atomic Energy (Radiation Protection) Rules 2004, Indian Rare Earths Limited (IREL), a wholly owned Public Sector Undertaking of the Government of India (GOI) under DAE, is the only entity which has been permitted to produce and process monazite, and handle it for domestic use as well as for export.

NOTE: In the question, in statement 3, the extreme word ‘entire’ is a red flag.

 

Question 5

Gandikota canyon of South India was created by which one of the following rivers?

(a) Cauvery

(b) Manjira

(c) Pennar

(d) Tungabhadra

Ans: c

Sub-Theme: Physical features of India

    • The gorge of Gandikota is situated on the Pennar River in Andhra Pradesh and is known as the Grand Canyon of India.
    • Gorges are formed when rivers erode through gigantic rock formations over a long period of time.
    • A gorge is any valley formed by a source of water flowing between tall slabs of steep
    • Gandikota is a small village in the Kadapa district of Andhra Pradesh.
    • The swift flowing waters of River Pennar cut through the granite rocks near the village of Gandikota, naturally eroding the rock and forming a deep valley.
    • The spectacular gorge is famously adjudged as the Grand Canyon of India.


 

Question 6

Consider the following pairs:

Peak Mountains
1. Namcha Barwa Garhwal Himalaya
2. Nanda Devi Kumaon Himalaya
3. Nokrek Sikkim Himalaya

Which of the pairs given above is/are correctly matched?

(a) 1 and 2

(b) 2 only

(c) 1 and 3

(d) 3 only

Ans: b        

Sub-Theme: Physical features of India

Pair 1 is incorrect: Namcha Barwa is not situated in Garhwal Himalaya. It is situated in the Eastern Himalaya.

Pair 2 is correct: The part of the Himalayas lying between Satluj and Kali rivers is known as Kumaon Himalayas. Nanda Devi is part of the Kumaon Himalayas, and is located in the state of Uttarakhand, between the Rishi Ganga valley on the west and the Goriganga valley on the east.

Pair 3 is incorrect: Nokrek is situated in West Garo Hills of Meghalaya, not in Sikkim Himalaya.

 

Question 7

Consider the following pairs:

Reservoirs States
1. Ghataprabha Telangana
2. Gandhi Sagar Madhya Pradesh
3. Indira Sagar Andhra Pradesh
4. Maithon Chhattisgarh

How many pairs given above are not correctly matched?

(a) Only one pair

(b) Only two pairs

(c) Only three pairs

(d) All four pairs

Ans: c          

Sub-Theme: Reservoir and their location

Pair 1 is incorrect: The Ghataprabha Reservoir is located in the Belagavi district of the state of Karnataka.

Pair 2 is correct: The Gandhi Sagar Dam is one of the four major dams built on India’s Chambal River located in the Mandsaur, districts of the state of Madhya Pradesh.

Pair 3 is incorrect: The Indira Sagar Dam is the largest dam in India, in terms of volume of water stored in the reservoir. It is located on the Narmada River at the town of Narmada Nagar, Punasa in the Khandwa district of Madhya Pradesh in India.

Pair 4 is incorrect: The Maithon Dam is constructed on the Barakar River and it is located at Maithon, 48 km from Dhanbad, in the state of Jharkhand in India.

 

2021

 

Question 1

Consider the following statements

  1. In the tropical zone, the western sections of the oceans are warmer than the eastern sections owing to the influence of trade winds.

  2. In the temperate zone, westerlies make the eastern sections of oceans warmer than the western sections.

Which of the statements given above is/are correct?

(a) 1 only

(b) 2 only

(c) Both 1 and 2

(d) Neither 1 nor 2

Ans: c

Sub-Theme: Wind Movement

Statement 1 is correct: The Trade winds originate from subtropical high pressure regions and move toward the equatorial low pressure belt. At the equator, the trade winds from the two hemispheres collide, rising and bringing torrential rainfall. The Northeast trade winds in the Northern hemisphere of the tropical zone move warmer water westward through the ocean. As a result of the cool ocean currents, the eastern regions of the trade winds are drier and more stable than the western parts of the ocean.

Statement 2 is correct: The westerlies are the winds blowing from the subtropical high pressure belts towards the sub polar low pressure belts.The Westerlies play an important role in carrying the warm, equatorial waters and winds to the western coasts of continents that is eastern section of the Oceans in the temperate zone.


 

Question 2

“Leaf litter decomposes faster than in any other biome and as a result the soil surface is often almost bare. Apart from trees, the vegetation is largely composed of plant forms that reach up into the canopy vicariously, by climbing the trees or growing as epiphytes, rooted on the upper branches of trees.”

This is the most likely description of

(a) Coniferous forest

(b) Dry deciduous forest

(c) Mangrove forest

(d) Tropical rainforest

Ans: d

Sub-Theme: Major Climate of the World

Hot, Wet Equatorial Climate:

  • Hot, Wet Equatorial Climate is found between 5° and 10° north and south of the equator.
  • Precipitation is heavy and well distributed throughout the year.
  • From the air, the tropical rain forest appears like a thick canopy of foliage, broken only where it is crossed by large rivers or cleared for cultivation.
  • All plants struggle upwards (most epiphytes) for sunlight resulting in a peculiar layer arrangement, rooted on the upper branches of trees.
  • The equatorial vegetation comprises a multitude of evergreen trees that yield tropical hardwood, g. mahogany, ebony, dyewoods etc.

2 1 1

NOTE: UPSC has this habit of picking lines from GC Leong and frame it as a question. Please learn to pick the hints from the UPSC question, here in the above sentence observe the keywords like ‘Leaf litter decomposes faster’; ‘canopy’; ‘epiphytes’; etc. these are enough/sufficient hints to answer this question. Now if you have read NCERTs and GC Leong you could easily identify that these are the characteristics of ‘Tropical Rainforest’ or ‘Hot, Wet Equatorial Climate’.

 

Question 3

The black cotton soil of India has been formed due to the weathering of:

(a) Brown forest soil

(b) Fissure volcanic rock

(c) Granite and schist

(d) Shale and limestone

Ans: b

Sub-Theme: Type of Soil

Option (b) is correct: The black cotton soils of India have been formed due to the weathering of the fissure volcanic rock.

Picture2 3

                                                                                                         Major Soils Types

 

Question 4

Among the following, which one is the least water- efficient crop?

(a) Sugarcane

(b) Sunflower

(c) Pearl millet

(d) Red gram

Ans: a

Sub-Theme: Water efficient crop

Option (a) is correct: Sugarcane requires around 1800 to 2200 mm and basically it is a heavy water intensive crop.

Option (b) is incorrect: Major oilseeds like soyabean, castor seeds, cotton seeds, linseed and sunflower, etc. requires around 672.4 mm.

Option (c) is incorrect: Millets can be grown even in dry regions without irrigation facilities. Pearl millets require around 350 mm of water.

Option (d) is incorrect: Pulses like red gram require around 200-450 mm of water.

 

Question 5

With reference to the Indus River system, of the following four rivers, three of them pour into one of them which joins the Indus directly. Among the following, which one is such a river that joins the Indus directly?

Chenab

(b) Jhelum

(c) Ravi

(d) Sutlej

Ans: d

Sub-Theme: Indus River System/Drainage

Option (a) is incorrect: The Chenab joins the Satluj in Panchnad(Pakistan) after receiving the waters of the Jhelum and Ravi rivers.

Option (b) is incorrect: The Jhelum joins the Chenab near Jhang in Pakistan.

Option (c) is incorrect: The Ravi debouches into the Chenab near Sarai Sidhu, a little above Rangpur in Pakistani Punjab.

Option (d) is correct: The Satluj receives the collective drainage of the Ravi, Chenab and Jhelum rivers. It joins the Indus a few kilometers above Mithankot.

Picture4 2

 

Question 6

With reference to India, Didwana, Kuchaman, Sargol and Khatu are the names of

(a) Glaciers

(b) Mangrove areas

(c) Ramsar sites

(d) Saline lakes

Ans: d

Sub-Theme: Physiography of India

  • A playa/salt lake or saline lake is a landlocked body of water that has a concentration of salts and other dissolved minerals significantly higher than most lakes. Didwana, Kuchaman, Sargol and Khatu are saline lakes of Rajasthan.

About Playas (Salt Lake):

  • Small lakes with flat floors and undrained basins in which water collects after rains and evaporates quickly are called playas., mostly saline.
  • Playas/Salt lake in Rajasthan: Sambhar, Didwana, Kuchaman, the Sargol and the Khatu

 

Question 7

Consider the following rivers:

  1. Brahmani

  2. Nagavali

  3. Subarnarekha

  4. Vamsadhara

Which of the above rise from the Eastern Ghats?

(a) 1 and 2

(b) 2 and 4

(c) 3 and 4

(d) 1 and 3

Ans: b

Sub-Theme: Peninsular Drainage (Small East Flowing Rivers)

Statement 1 is incorrect: The Brahmani river comes into existence by the confluence of the Koel and the Shankh rivers near Rourkela.

Statement 2 is correct: Nagavali River originates in Kalahandi District, in the Eastern Ghats.

Statement 3 is incorrect: The Subarnarekha originates from the Ranchi Plateau in Jharkhand forming the boundary between West Bengal and Odisha in its lower course.

Statement 4 is correct: It is an east-flowing river that originates in the Kalahandi district of Odisha, Eastern Ghat, flows in Odisha, along its boundary with Andhra Pradesh and finally joins the Bay of Bengal at Kalingapatnam, Andhra Pradesh.

 

2020

 

Question 1

Consider the following statements

  1. Jet streams occur in the Northern Hemisphere only.

  2. Only some cyclones develop an eye.

  3. The temperature inside the eye of a cyclone is nearly 10°C less than that of the surroundings.

Which of the statements given above is/are correct?

(a) 1 only

(b) 2 and 3 only

(c) 2 only

(d) 1 and 3 only

Ans: c

Sub-Theme: Wind Movement

Jet Stream:

  • The Jet Stream is a geostrophic wind that generally blows from west to east across the upper layers of the troposphere at an altitude of 20,000 to 50,000 feet.
  • Air masses with different temperatures collide to form jet streams. Thus, the location of the Jet Stream’s formation is normally characterized by surface temperatures.

Statement 1 is not correct: In both the hemispheres, jet streams run from 20 degrees latitude to the poles.

Statement 2 is correct: Only some cyclones develop an eye. The eye is not present in a temperate cyclone since there is not a single location where the winds and precipitation are not present.

Statement 3 is not correct: The tropical cyclone’s eye is the area with the lowest surface pressure and the warmest air above it (in the upper levels). The eye temperature may be 10°C or more above the surrounding air at a height of 12 km, whereas it is only 0–2°C warmer at the surface.

 

Question 2

With reference to Ocean Mean Temperature (OMT), which of the following statements is/are correct?

  1. OMT is measured up to a depth of 26°C isotherm which is 129 metres in the south-western Indian Ocean during January–March.

  2. OMT collected during January–March can be used in assessing whether the amount of rainfall in monsoon will be less or more than a certain long-term mean.

Select the correct using the code given below:

(a) 1 only

(b) 2 only

(c) Both 1 and 2

(d) Neither 1 nor 2

Ans: b

Sub-Theme: Oceanography

About Ocean Mean Temperature (OMT):

  • It is analysed by measuring the ocean thermal energy between January and March, and it is more accurate than SST at forecasting the Indian summer monsoon.
  • Unlike SST, OMT was able to accurately predict whether the summer monsoon rainfall was greater than or less than the long-term average in 20 of 25 years (80% success rate).
  • The reason why OMT performs better than SST is because OMT better represents the upper ocean thermal energy And the variations in the upper ocean thermal energy conditions are mainly responsible for the summer monsoon

Statement 1 is incorrect: OMT is measured up to a depth of 26°C isotherm at depths varying from 50-100 meters not 129 meters

Statement 2 is correct: OMC is analyzed by measuring the ocean thermal energy during the January-March period – can better predict Indian summer monsoon than the SST.

 

Question 3

Consider the following minerals:

  1. Bentonite

  2. Chromite

  3. Kyanite

  4. Sillimanite

In India, which of the above is/are officially designated as major minerals?

(a) 1 and 2 only

(b) 4 only

(c) 1 and 3 only

(d) 2, 3 and 4 only

Ans: d

Sub-Theme: Major Minerals in India

  • Chromite, Kyanite  and  Sillimanite  are metallic minerals and metallic minerals mostly come in the major mineral category. Therefore, the correct answer is (d) that is 2, 3 and 4 only.
  • Major minerals: Major minerals are those specified in  the  Mines  and  Minerals (Development and Regulation) Act, 1957. Major minerals include: Lignite, Chromite, Uranium, Kyanite, Coal, Gold, Iron ore, Lead-Zinc, Magnesium, Sillimanite, Tungsten, and Diamond etc.
  • Minor minerals: Here are few minerals that the Central Government has declared as minor minerals: boulder, sand, shingle, chalcedony pebbles, lime shell, kankar and limestone, brick-earth, fuller’s earth, bentonite, road metal, slate, marble, stone used for making household utensils, quartzite and sandstone, saltpetre and ordinary earth.

 

Question 4

“The crop is subtropical in nature. A hard frost is injurious to It requires at least 210 frost-free days and 50 to 100 centimeters of rainfall for its growth. A light well-drained soil capable of retaining moisture is ideally suited for the cultivation of the crop.”

Which one of the following is that crop?

(a) Cotton

(b) Jute

(c) Sugarcane

(d) Tea

Ans: a         

Sub-Theme: Major Crop

Cotton:

  • Temperature: Between 21-30°C
  • Rainfall: Around 50-100 cm.
  • Soil Type: Well drained black cotton soil of Deccan Plateau.
  • Top Cotton Producing States: Gujarat > Maharashtra > Telangana > Andhra Pradesh > Rajasthan.
  • It is believed that the cotton plant originated in India. One of the primary raw materials used in the cotton textile business is cotton.
  • Cotton needs 210 frost-free days and bright sun-shine for its growth.
  • It is a kharif crop and requires 6 to 8 months to mature.
  • The government of India has launched the Silver Fibre Revolution and Technology Mission on Cotton programmes to increase cotton production.
  • For protection against pest attacks and environmental stress, cotton has been genetically altered into BT Cotton.

NOTE: Learn to pick keywords/keyphrases from these long sentences to reach the correct answer. Here the keywords are – ‘subtropical’; ‘210 frost-free days’; ‘50 to 100 centimeters of rainfall’; ‘well- drained soil’. These keywords/keyphrases are enough to mark the correct answer. Also, this underlines the importance of reading NCERTs. UPSC asks questions on various crop/crops and their characteristics every alternate year. It is advisable to read India People and Economy class XII NCERT thoroughly.

 

Question 5

Which of the following Protected Areas are located in Cauvery basin?

  1. Nagarhole National park

  2. Papikonda National Park

  3. Sathyamangalam Tiger Reserve

  4. Wayanad Wildlife Sanctuary

Select the correct answer using the code given below:

(a) 1 and 2 only

(b) 3 and 4 only

(c) 1, 3 and 4 only

(d) 1, 2, 3 and 4

Ans: c

Sub-Theme: Major Places and Rivers/Map based

Option 1 is correct: The Nagarahole River flows through the park, which joins the Kabini River which also is a boundary between Nagarahole and Bandipur National Park. Kabini, a tributary of the Cauvery River, is the largest river draining the park.

Option 2 is incorrect: The Papikonda park lies on the left and right banks of the river Godavari and cuts through the Papikonda hill range of Eastern Ghats.

Option 3 is correct: Sathyamangalam Tiger Reserve (STR) is located at the confluence region of Western and Eastern Ghats, in the Erode district of the state of Tamil Nadu. In the northern part of Erode district, the Palar river flows and drains into the Cauvery river.

Option 4 is correct: Kabini river, a tributary of the Cauvery river flows through the Wayanad Wildlife Sanctuary.

 

Question 6

Siachen Glacier is situated to the:

(a) East of Aksai Chin

(b) East of Leh

(c) North of Gilgit

(d) North of Nubra Valley

Ans: d

Sub-Theme: Map based

The Siachen Glacier is situated to the North of the Nubra Valley.

About Siachen Glacier:

Siachen Glacier is located in the eastern Karakoram range in the Himalayas at about 421226°N 77.109540°E, just northeast of the point NJ9842 where the Line of Control between India and Pakistan ends. The 75 km long Siachen Glacier in the north of Nubra valley has the distinction of being the largest glacier outside the polar and the subpolar regions. It is also the world’s highest battlefield.

Picture5

NOTE: Siachen Glacier was in the news. This is purely a map based question and anyone with a little bit of knowledge can easily solve this question. Observing the map carefully, East of Aksai Chin is China Administered Tibet, while Leh is situated South of Siachen, and Gilgit is on the western side of Siachen.

 

Question 7

Consider the following pairs:

River Flow into
1. Mekong Andaman Sea
2. Thames Irish Sea
3. Volga Caspian Sea
4. Zambezi Indian Ocean

Which of the pairs given above is/are correctly matched?

(a) 1 and 2 only

(b) 3 only

(c) 3 and 4 only

(d) 1, 2 and 4 only

Ans: c

Sub-Theme: Map based

Statement 1 is incorrect: The Mekong River originates in the icy headwaters of the Tibetan highlands. It flows through the steep canyons of China, known as the upper basin, through lower basin countries Myanmar, Laos, Thailand, and Cambodia, before fanning an expansive delta in Vietnam and emptying into the South China Sea.

Statement 2 is incorrect: It is the longest river in England, flowing 215 miles from the Cotswolds to the North Sea. The main tributaries of Thames are Buscot, Reading, and Kingston.

52 1

Statement 3 is correct: The Volga River, the longest river in Europe, runs through Russia with its delta flowing into the Caspian Sea just south of the Kazakhstan border.

Statement 4 is correct: The Zambezi is the fourth-largest river after the Congo/Zaire, Nile and Niger in Africa. It rises in the Kalene hills in north-western Zambia and flows eastwards for about 3000 km to the Indian Ocean.

 

Question 8

With reference to pulse production in India, consider the following statements:

  1. Black gram can be cultivated as both kharif and rabi crops.

  2. Green-gram alone accounts for nearly half of pulse

  3. In the last three decades, while the production of kharif pulses has increased the production of rabi pulses has decreased.

Which of the statements given above is/are correct?

(a) 1 only

(b) 2 and 3 only

(c) 2 only

(d) 1, 2 and 3

Ans: a          

Sub-Theme: Major crops

Statement 1 is correct: In India, the important pulse crops grown in winter (rabi) are chickpea, lentil, lathyrus, field pea and kidney bean. However, green gram, black gram and cowpea are grown in both spring and rainy season.

Statement 2 is incorrect: According to the Directorate of Economics and Statistics (DES), the share of pulse production in 2018- 19 was comprised of Tur (15.34%), Gram (43.29%), Moong (green gram,10.04%), Urad (black gram, 13.93%), Lentil (6.67%), and Other Pulses (10%).

Statement 3 is incorrect: In the last three decades, both, the production of kharif pulses and the production of rabi pulses have increased.

 

2019

 

Question 1

On 21st June, the Sun:

(a) Does not set below the horizon at the Arctic Circle

(b) Does not set below the horizon at Antarctic Circle

(c) Shines vertically overhead at noon on the Equator

(D) Shines vertically overhead at the Tropic of Capricorn

Ans: a

Sub-Theme: Motion of Earth

Option (a) is correct: Areas near the poles receive less heat as the rays of the sun are slanting.The North Pole is inclined towards the sun and the places beyond the Arctic Circle experience continuous daylight for about six months.

Summer Solstice (21st June):

  • The Northern Hemisphere is tilted towards the sun and the rays of the sun fall directly on the Tropic of Cancer hence the areas receive more heat.

1

Revolution of the Earth and Seasons

  • Since a large portion of the Northern Hemisphere is getting light from the sun, it is summer in the regions north of the equator. The longest day and the shortest night at these places occur on 21st June.
  • Southern Hemisphere: All these conditions are reversed (winter season- longer nights)

 

Question 2

Why are dewdrops not formed on a cloudy night?

(a) Clouds absorb the radiation released from the Earth’s surface.

(b) Clouds reflect back the Earth’s radition.

(c) The Earth’s surface would have low temperatures on cloudy nights.

(d) Clouds deflect the blowing wind to ground level

Ans: b

Sub-Theme: Dew Formation/Water in Atmosphere

Option (b) is correct: The dew drops are not formed on a cloudy night because the Clouds reflect back the Earth’s radiation.

  • Cloud cover prevents radiation from earth surfaces from escaping during cloudy nights. It maintains a high air temperature. Additionally, the moisture won’t condense and turn into dew drops on the grass and leaves. Dew is formed when the air temperature touches the dew point.
  • Dew point is the temperature at which air becomes saturated with water vapor, causing condensation to form. It is the temperature at which the air would need to be cooled in order for the relative humidity to reach 100%.

 

Question 3

With reference to the management of minor minerals in India, consider the following statements:

  1. Sand is a ‘minor mineral’ according to the prevailing law in the country.

  2. State Governments have the power to grant mining leases of minor minerals, but the powers regarding the formation of rules related to the grant of minor minerals lie with the Central government.

  3. The State Government has the power to frame rules to prevent illegal mining of minor minerals.

Which of the statements given above is/are correct?

(a) 1 and 3 only

(b) 2 and 3 only

(c) 3 only

(d) 1, 2 and 3

Ans: a      

Sub-Theme: Minor Minerals in india

Statement 1 is correct: Sand is a ‘minor mineral’ according to the prevailing law in the country.

Statement 2 is incorrect: Any mineral which by the notification of the Central Government may declare to be a minor mineral.

Statement 3 is correct: The State Government may, by notification in the Official Gazette, make rules for regulating the grant of quarry leases, mining leases or other mineral concessions in respect of minor minerals and for purposes connected there with.

 

Question 4

With reference to the cultivation of Kharif crops in which in the last five years, consider the following statements:

  1. Area under rice cultivation is the highest.

  2. Area under cultivation of jowar is more than that of oilseeds.

  3. Area of cotton cultivation is more than that of

  4. Area under sugarcane cultivation has steadily

Which of the statements given above is/are correct?

(a) 1 and 3 only

(b) 2, 3 and 4 only

(c) 2 and 4 only

(d) 1, 2, 3 and 4

Ans: a

Sub-Theme: Major Crops

Statement 1 is correct: The area under rice cultivation in India, in 2013-14 was 44.13 million hectares while in 2015-16, it became 43.39 million hectares and is highest amongst all.

Statement 2 is incorrect: The annual area under Jowar ranges between 17 and 18 million hectares while the oilseed area is 28 million hectares (2013-14), 26.1 million hectares (2015-16) i.e area under the cultivation of Jowar is less than that of oilseeds.

Statement 3 is correct: The area under sugarcane cultivation is 4.99 million hectares (2013-14), 5.066 million hectares (2014-15), 4.953 million hectares (2015-16). The area under cotton cultivation is 11.96 million hectares (2013-14), 12.81 million hectares (2014-15), and 11.87 million hectares (2015-16). Hence the area under cotton cultivation is more than sugar cultivation.

Statement 4 is incorrect: The area under sugarcane cultivation has not steadily decreased.

 

Question 5

What is common to the places known as Aliyar, Isapur and Kangsabati?

(a) Recently discovered uranium deposits

(b) Tropical rain forests

(c) Underground cave systems

(d) Water reservoirs

Ans: d

Sub-Theme: Current Affairs

Aliyar, Isapur and Kangsabati are the names which have water reservoirs common to the people.

About Water Reservoirs:

  • In Tamil Nadu’s Coimbatore district, the Aliyar reservoir is a body of water near Pollachi town.
  • In the Maharashtra district of Hingoli, there is an earth-fill dam called the Isapur Dam on the Penganga River.
  • As a part of the Indian Second Five-Year Plan, the Kangsabati Reservoir Project was launched in 1956 to provide water to the districts of Midnapur, Bankura, and It involves irrigation of land using water from the Kangsabati River, as well as the Shilabati and the Bhoirobbanki rivers.

 

Question 6

Consider the following pairs:

Famous Place

River

1. Pandharpur Chandrabhaga
2. Tiruchirappalli Cauvery
3. Hampi Malaprabha

Which of the pairs given above are correctly matched?

(a) 1 and 2 only

(b) 2 and 3 only

(c) 1 and 3 only

(d) 1, 2 and 3

Ans: a

Sub-Theme: Famous places of India

Pair 1 is correct: Pandharpur city is on the banks of Chandrabhaga River in Maharashtra. The famous Lord Vitthal-Rukmini Mandir is located in this city.

Pair 2 is correct: Tiruchirappalli is an Indian city of Tamil Nadu on the banks of river Cauvery. The famous Sri Ranganathaswamy Temple is located in Tiruchirappalli.

Pair 3 is incorrect: Hampi was the capital of the Vijayanagara kingdom and is located on the bank of the Tungabhadra River in present state of Karnataka. Many famous temples like Virupaksha Temple, Nandi Statue etc are located in Hampi. Pattadakal or Pattadakal is situated on the banks of the river Malaprabha and is located in Karnataka.

 

Question 7

Consider the following pairs:

Sea

Bordering Country

1. Adriatic Sea Albania
2. Black Sea Croatia
3. Caspian Sea Kazakhstan
4. Mediterranean Sea Morocco
5. Red Sea airyS

Which of the pairs given above are correctly matched?

(a) 1, 2 and 4 only

(b) 1, 3 and 4 only

(c) 2 and 5 only

(d) 1, 2, 3, 4 and 5

Ans: b

Sub-Theme: World Map

Pair 1 is correct: The Adriatic Sea is a body of water separating the Italian Peninsula from the Balkan peninsula. The countries with coasts on the Adriatic sea: Albania, Bosnia and Herzegovina, Croatia, Italy, Montenegro and Slovenia. Mnemonic: ISC-MBA (Italia, Slovenia, Croatia, Montenegro, Bosnia, Albania).

1 1

Pair 2 is incorrect: Black Sea is a large inland sea situated at the southeastern extremity of Europe. It is bordered by Ukraine to the north, Russia to the northeast, Georgia to the east, Turkey to the south, and Bulgaria and Romania to the west. Black Sea is not bordered by Croatia. Mnemonic: Tea & BURGeR (Turkey, Bulgaria, Ukraine, Russia, Georgia, Romania).

2 2

Pair 3 is correct: Caspian Sea is the world’s largest inland body of water, variously classed as the world’s largest lake or a full-fledged sea. It is an endorheic basin (a basin without outflows) located between Europe and Asia. It is bounded by Kazakhstan to the northeast, Russia to the northwest, Azerbaijan to the west, Iran to the south, and Turkmenistan to the southeast. Mnemonic: TARIK (Turkmenistan, Azerbaijan, Russia, Iran, Kazakhstan).

3 2

Pair 4 is correct: The Mediterranean Sea is an intercontinental sea that stretches from the Atlantic Ocean on the west to Asia on the east and separates Europe from Africa. The countries surrounding the Mediterranean in clockwise order: Spain, France, Monaco, Italy, Slovenia, Croatia, Bosnia and Herzegovina, Montenegro, Albania, Greece, Turkey, Syria, Lebanon, Israel, Egypt, Libya, Tunisia, Algeria, and Morocco.

4

Pair 5 is incorrect: Red Sea is a narrow strip of water extending south-eastward from Suez, Egypt, for about 1,930 km to the Bab-el-Mandeb Strait, which connects with the Gulf of Aden and thence with the Arabian Sea. The six countries bordering the Red Sea: Saudi Arabia, Yemen, Egypt, Sudan, Eritrea and Djibouti. The Red Sea is not bordered by Syria. Mnemonic: DESSEY (Djibouti, Eritrea, Saudi Arabia, Sudan, Egypt, Yemen).

4 5

 

2018

 

Question 1

Consider the following statements

  1. Most of the world’s coral reefs are in tropical.

  2. More than one-third of the world’s coral reefs are located in the territories of Australia, Indonesia and Philippines.

  3. Coral reefs host far more animal phyla than those hosted by tropical rainforests.

Which of the statements given above is/are correct?

(a) 1 and 2 only

(b) 3 only

(c) 1 and 3 only

(d) 1, 2 and 3

Ans: d

Sub-Theme: Coral reef distribution

    • Coral reefs are underwater ecosystems formed by colonies of tiny animals called coral polyps. These animals secrete a hard, calcium carbonate exoskeleton that provides a foundation for the growth of other corals and a diverse array of marine organisms. Coral reefs are found in tropical and subtropical waters around the world, particularly in the Pacific Ocean, the Indian Ocean, and the Caribbean Sea.

Statement 1 is correct: The majority of reef- building corals are found within tropical and subtropical waters. These typically occur between 30° north and 30° south latitudes.

6 1 Indian Ocean Dipole (IOD): Neutral Phase

Statement 2 is correct: The Indonesian/ Philippines archipelago, Great Barrier Reef of Australia, the Red Sea, and the Caribbean has the world’s greatest concentration of reefs and the greatest coral diversity.

Statement 3 is correct: Some scientists estimate that more than 25,000 described species from thirty-two of the world’s thirty-three animal phyla live in reef habitats four times the number of animal phyla found in tropical rainforests.

 

Question 2

Consider the following statements:

  1. In India, State Governments do not have the power to auction non-coal mines.

  2. Andhra Pradesh and Jharkhand do not have gold mines.

  3. Rajasthan has iron ore mines.

Which of the statements given above is/are correct?

(a) 1 and 2

(b) 2 only

(c) 1 and 3

(d) 3 only

Ans: d

Sub-Theme: Mines and Mineral/Mineral Rules

Statement 1 is incorrect: As per the MMDR Amendment Act 2015, the state governments will conduct auctions for grant of mineral concessions. The role of the central government is to prescribe the terms and conditions and procedures subject to which the auction shall be conducted.

Statement 2 is incorrect: Largest gold ore (primary) are located in Bihar (44%) followed by Rajasthan (25%) and Karnataka (21%), West Bengal, and Andhra Pradesh (3% each).

Statement 3 is correct: Hematite and magnetite are the most important iron ores in India and resources of hematite are spread in Andhra Pradesh, Assam, Bihar, Maharashtra, Madhya Pradesh, Meghalaya, Rajasthan and Uttar Pradesh.

 

Question 3

Which one of the following is an artificial lake?

(a) Kodaikanal (Tamil Nadu)

(b) Kolleru (Andhra Pradesh)

(c) Nainital (Uttarakhand)

(d) Renuka (Himachal Pradesh)

Ans: a

Sub-Theme: Important Lakes of India

About Kodaikanal Lake:

  • Kodaikanal Lake, also known as Kodai Lake is a manmade lake (artificial) located in the Kodaikanal city in Dindigul district in Tamil Nadu, India.
  • The lake was built in 1863, in the middle of Kodaikanal town, which was established by the British and early American missionaries. Sir Vere Henry Levinge, the then Collector of Madurai, had a key role in its
  • The lake is star-shaped, centrally located in the town of Kodaikanal and is surrounded by lush green hills of the northwestern Palani Hills range, which is the main watershed for the lake.

 

Question 4

Consider the following statements:

  1. The Barren Island volcano is an active volcano located in Indian territory.

  2. Barren Island lies about 140 km east of Great Nicobar.

  3. The last time the Barren Island volcano erupted was in 1991 and it has remained inactive since then.

Which of the statements given above is/are correct?

(a) 1 only

(b) 2 and 3

(c) 3 only

(d) 1 and 3

Ans: a

Sub-Theme: Physiography of India

Statement 1 is correct: Barren Island Volcano is a part of Indian Union Territory of Andaman and Nicobar islands is an active volcano in South Asia (along a chain of volcanoes from Sumatra to Myanmar).

Statement 2 is incorrect: Barren Island is located in Andaman Sea, about 140 km northeast of Port Blair.

Statement 3 is incorrect: The last time the volcanic eruptions were reported and linked to 28 September 2018 earthquakes in Sulawesi, Indonesia.

NOTE: In 2015 UPSC has asked a question on Andaman & Nicobar Island, so for detailed information on A&N Islands please refer to the 2015 PYQ.

 

Question 5

Which of the following has/have shrunk immensely/ dried up the recent past due to human activities?

  1. Aral Sea

  2. Black Sea

  3. Lake Baikal

Select the correct answer using the code given below:

(a) 1 only

(b) 2 and 3

(c) 2 only

(d) 1 and 3

Ans: a

Sub-Theme: World Map/Environment and Ecology

About Aral Sea:

  • The Aral Sea, a salt lake in Central Asia, was previously the fourth-largest salt lake in the world..
  • Drainage basin: Kyrgyzstan, Turkmenistan, Tajikistan, Uzbekistan, Pakistan, Afghanistan, and Kazakhstan.
  • The Aral Sea started to shrink in the 1960s as a result of the Soviet Union starting to divert water for agricultural use.
  • According to NASA, water diversion for agricultural purposes in the region has caused the Aral Sea to lose nearly 75% of its original size during the past 50 years.
  • The Aral Sea’s depth has similarly dropped, going from 68 metres in the 1960s to less than 10 meters now.

7

 

Question 6

Among the following cities, which one lies on a longitude closest to that of Delhi?

(a) Bengaluru

(b) Hyderabad

(c) Nagpur

(d) Pune

Longitude of Indian Cities

  • The longitude of Delhi is 77.216721°E
  • The longitude of Bangalore is 77.59450 E.
  • The longitude of Nagpur is 79.0888° E.
  • The longitude of Pune is 73.8474” E.

8

NOTE: Map Practicing will not only help you to answer such questions but also it will help you to fetch some extra marks in UPSC Mains as well.

 

Question 7

Consider the following pairs:

Regions sometimes mentioned in news Country
1. Catalonia Spain
2. Crimea Hungary
3. Mindanao Philippines
4. Oromia Nigeria

Which of the pairs given above are correctly matched?

(a) 1, 2 and 3

(b) 3 and 4 only

(c) 1 and 3 only

(d) 2 and 4 only

Ans: c

Sub-Theme: Places was in News

Pair 1 is correct: Catalonia is located in Spain bordered by France and Andorra in the north, the Mediterranean Sea to the east, the autonomous community of Valencia to the south, and the autonomous community of Aragon to the west.

10

Pair 2 is incorrect: The Republic of Crimea, officially part of Ukraine, lies on a peninsula stretching from the south of Ukraine between the Black Sea and the Sea of Azov. It is separated from Russia by the narrow Kerch Strait.

11

Pair 3 is correct: Mindanao is an island located in the Philippines. It is surrounded by the Bohol, Philippine, Celebes, and Sulu seas.

78

Pair 4 is incorrect: Oromia is a regional state of Ethiopia. Also, it is to be remembered that Ethiopia is a landlocked country located in Eastern Africa.

67 1

 

Question 8

Consider the following pairs:

Towns sometimes mentioned in news Country
1. Aleppo Syria
2. Kirkuk Yemen
3. Mosul Palestine
4. Mazar-i-sharif Afghanistan

Which of the pairs given above are correctly matched?

(a) 1 and 2

(b) 1 and 4

(c) 2 and 3

(d) 3 and 4

Ans: b

Sub-Theme: Places in News

Name of Cities and Countries:

  • Aleppo is in Syria
  • Kirkuk is in Iraq.
  • Mosul in Iraq
  • Mazar-i-sharif is in Afghanistan, Kabul (Capital of Afghanistan)

 

2017

 

Question 1

With reference to ‘Indian Ocean Dipole (IOD)’ sometimes mentioned in the news while forecasting Indian monsoon, which of the following statements is/are correct?

  1. The IOD phenomenon is characterised by a difference in sea surface temperature between tropical Western Indian Ocean and tropical Eastern Pacific Ocean.

  2. An IOD phenomenon can influence an El Nino’s impact on the monsoon.

Select the correct answer using the code given below:

(a) 1 only

(b) 2 only

(c) Both 1 and 2

(d) Neither 1 nor 2

Ans: b

Sub-Theme: Indian Ocean Dipole

Statement 1 is incorrect: IOD is the difference in sea surface temperature between two poles or areas i.e. an eastern pole in the eastern Indian Ocean south of Indonesia, and a western pole in the Arabian Sea (western Indian Ocean).

Statement 2 is correct: Negative IOD events are often associated with La Nina and positive IOD events with El Nino.

 

Question 2

At one of the place in India, if you stand on the seashore and watch the sea, ‘you will find that the sea water recedes from the shore line a few kilometres and comes back to the shore, twice a day, and you can actually walk on the seafloor when the water recedes.’ This unique phenomenon is seen at

(a)Bhavnagar

(b) Bheemunipatnam

(c) Chandipur

(d) Nagapattinam

Ans: c

Sub-Theme: Basic Trivia

About Chandipur Sea Beach:

  • A unique phenomenon rarely seen anywhere else, the sea recedes by as much as five kilometres every day on the Chandipur beach in eastern India.
  • The sea recedes massively from the beach during ebb and returns to fill the emptiness during high tide.
  • This hide-and-seek of the sea comes to play twice a day and seeing the sea disappear and then return is an unforgettable experience. Thus it is also called the ‘hide-and-seek’ beach.
  • Chandipur also has the DRDO (Defence Research and Development Organization) Integrated Research Range. In this range ballistic missiles are test-fired.

 

Question 3

With reference to river Teesta, consider the following statements:

1. The source of river Teesta is the same as that of Brahmaputra but it flows through Sikkim.

2. River Rangeet originates in Sikkim and it is a tributary of river Teesta.

3. River Teesta flows into Bay of Bengal on the border of India and Bangladesh.

Which of the statements given above is/are correct?

(a) 1 and 3 only

(b) 2 only

(c) 2 and 3 only

(d) 1, 2 and 3

Ans: b

Sub-Theme: Himalayan River/Drainage

Statement 1 is incorrect: Teesta originates in the Himalayas near Chunthang, Sikkim and flows to the south through West Bengal before entering Bangladesh.

Statement 2 is correct: River Rangeet originates in Sikkim, which is the largest river of Sikkim and also a tributary of river Teesta.

Statement 3 is incorrect: Originally, the river continued southward to empty directly into the Padma River (the main channel of Ganga in Bangladesh) but around 1787 the river changed its course to flow eastward to join the Jamuna river.

 

Question 4

Mediterranean Sea is a border of which of the following countries?

  1. Jordan

  2. Iraq

  3. Lebanon

  4. Syria

Select the correct answer using the code given below:

(a) 1, 2 and 3 only

(b) 2 and 3 only

(c) 3 and 4 only

(d) 1, 3 and 4 only

Ans: c

Sub-Theme: World Map

Mediterranean Sea Bordering countries:

  • The countries bordering the Mediterranean Sea are Albania, Algeria, Bosnia and Herzegovina, Croatia, Cyprus, Egypt, France, Greece, Israel, Italy, Lebanon, Libya, Malta, Monaco, Montenegro, Morocco, Slovenia, Spain, Syria, Tunisia, and Turkey.

1 4

 

Question 5

Which of the following is geographically closest to Great Nicobar?

(a) Sumatra

(b) Borneo

(c) Java

(d) Sri Lanka

Ans: a

Sub-Theme: Indian Ocean Region

About Great Nicobar:

  • Great Nicobar lies on the south of the Nicobar Islands of India and it is the largest island of the Nicobar South of Great Nicobar is the island of Sumatra.
  • Great Nicobar is closest to The distance between the two places is 1,192 kms.
  • The distance from Great Nicobar to Borneo is 2,398 kms.
  • The distance from Great Nicobar to Java stands at 1,817 kms.

2 2 1

 

Question 6

If you travel by road from Kohima to Kottayam, what is the minimum number of States within India through which you can travel, including the origin and the destination?

(a) 6

(b) 7

(c) 8

(d) 9

Ans: b

Sub-Theme: India Map

    • If travelling by road from Kohima to Kottayam, you need to cross Kohima the capital of Nagaland (origin), Assam, West Bengal, Odisha, Andhra Pradesh and then through either Tamil Nadu to Kerala (Kottayam-destination), or through AP to Karnataka to Kerala (Kottayam- destination). So the minimum number of States within India through which you can travel, including the origin and the destination is seven (7).

3 2 1

 

Question 7

Consider the following statements:

  1. In India, the Himalayas are spread over five states only.

  2. Western Ghats are spread over five states

  3. Pulicat Lake is spread over two States only.

Which of the statements given above is/are correct?

(a) 1 and 2 only

(b) 3 only

(c) 2 and 3 only

(d) 1 and 3 only

Ans: b

Sub-Theme: India Map

Statement 1 is incorrect: The Himalayas is spread across 12 Indian States: Jammu & Kashmir, Uttarakhand, Himachal Pradesh, Arunachal Pradesh, Manipur, Meghalaya, Mizoram, Nagaland, Sikkim, Tripura, two districts of Assam namely Dima Hasao and Karbi Anglong and Darjeeling and Kalimpong in West Bengal).

Statement 2 is incorrect: The Western Ghats are spread over six states namely Kerala, Tamil Nadu, Karnataka, Goa, Maharashtra and Gujarat.

4 2

Statement 3 is correct: Pulicat Lake is the second largest brackish water lake in India and spreads over the two states namely Andhra Pradesh and Tamil Nadu. Large varieties of birds, including painted storks and Grey pelicans, frequent the area every year. Grey Pelican and Painted Stork both are near-threatened species under IUCN Red List of Threatened Species.

6 1 2

 

 

2016

 

Question 1

Consider the following pairs:

Famous Place Region
1. Bodhgaya Baghelkhand
2. Khajuraho Bundelkhand
3. Shirdi Vidarbha
4. Nasik (Nashik) Malwa
5. Tirupati Rayalaseema

Which of the pairs given above are correctly matched? (a) 1, 2 and 4

(b) 2, 3, 4 and 5

(c) 2 and 5 only

(d) 1, 3, 4 and 5

Ans: c

Sub-Theme: Religious Places

Pair 1 is incorrect: Bodh Gaya is a holy landmark and pilgrimage destination linked with the Mahabodhi Temple Complex in Gaya, Bihar, India. It is one of the four main pilgrimage sites related with the life of Gautama Buddha, the other three are Kushinagar, Lumbini, Sarnath. In 2002, UNESCO declared the Mahabodhi Temple in Bodh Gaya a World Heritage Site.

Pair 2 is correct: The Khajuraho Temples in Madhya Pradesh’s Bundelkhand region are some of the most exquisite examples of medieval architecture in the world. It was built by the Chandela Dynasty between 950 and 1050 AD. There are Hindu and Jain temples among the monuments. In 1986, UNESCO designated these temples as World Heritage Sites. The temples are renowned for the symbolic architecture in the Nagara style. Initially reported in 1022 AD by Abu Raihan al Biruni and in AD 1335 by Ibn Battuta.

Pair 3 is incorrect: Shirdi is a city located in the Rahata taluka of Ahmednagar District, Maharashtra. According to legend, Sai Baba arrived in Shirdi in 1872 and remained there until his death on October 15, 1918.

Pair 4 is incorrect: Nashik is situated on the banks of river Godavari in the northwest region of Maharashtra and is famous for Kumbh Mela, which is held every 12 years.

Pair 5 is correct: Tirupati is a city situated in the Rayalaseema region of the Chittoor district of the Indian state of Andhra Pradesh. Shri Venkateswara Temple is well-known for being a Hindu temple.

 

Question 2

Which of the following is/are tributary/ tributaries of Brahmaputra?

  1. Dibang

  2. Kameng

  3. Lohit

Select the correct answer using the code given below.

(a) 1 only

(b) 2 and 3 only

(c) 1 and 3 only

(d) 1, 2 and 3

Ans: d

Sub-Theme: Himalayan Drainage System/ Brahmaputra

Option 1 and 3 are correct: Dhansiri, Dibang and Lohit are the left bank tributaries of Brahmaputra River.

Option 2 is correct: Kameng is a right bank tributary of Brahmaputra River. Other major right bank tributaries of Brahmaputra are Subansiri, Manas, Sankosh.

8 1

 

Question 3

In which of the following regions of India are shale gas resources found?

  1. Cambay Basin

  2. Cauvery Basin

  3. Krishna-Godavari Basin

Select the correct answer using the code given below.

(a) 1 and 2 only

(b) 3 only

(c) 2 and 3 only

(d) 1, 2 and 3

Ans: d

Sub-Theme: Resource/Map

About Shale Gas in India:

  • The Ministry of Petroleum and Natural Gas (MOPNG) has identified six basins as potentially shale gas bearing.

Picture5 1

  • These are: Cambay, Assam-Arakan, Gondwana, Krishna-Godavari Basin, Cauvery Basin and the Indo Gangetic
  • Fine-grained sedimentary rocks called shale are potential rich sources of natural gas and
  • Shale rocks are typically found next to ‘aquifers’, or rocks that contain usable/ drinking water.
  • Shale gas is trapped under low permeable

 

2015

 

Question 1

In the South Atlantic and South-Eastern Pacific regions in tropical latitudes, cyclone does not What is the reason?

(a) Sea surface temperatures are low

(b) Inter-tropical Convergence Zone seldom occurs

(c) Coriolis force is too weak

(d) Absence of land in those regions

Ans: b

Sub-Theme: Wind Movement

Option (b) is correct: The absence of cyclones can be demonstrated by the seldom occurance of ITCZ Convergence zone in the south east Pacific and south Atlantic regions. Due to this there is no creation of ideal conditions for Tropical Cyclone, for e.g. instead of low, there is high vertical wind sheer.

Cyclone:

  • A cyclone is a weather phenomenon characterized by a low-pressure center surrounded by a rotating system of winds that spiral inward.

Classification of Cyclone:

  • Tropical Cyclone
  • Extra tropical  cyclone  also  called

Temperate cyclone

Factors Responsible for for Tropical Cyclone Formation:

  • Warm Ocean Water: Cyclones form over warm ocean water, typically with temperatures of 26 degrees Celsius or higher. This warm water provides the energy that fuels the storm.
  • Low Atmospheric Pressure: Cyclones form in areas where the atmospheric pressure is low. This creates an area of low pressure that draws in warm, moist air from the surrounding environment.
  • Coriolis Effect: The Coriolis effect is a phenomenon caused by the rotation of the Earth. It causes air to be deflected to the right in the Northern Hemisphere and to the left in the Southern Hemisphere. This effect is what causes cyclones to spin in a clockwise direction in the Southern Hemisphere and counterclockwise in the Northern Hemisphere.
  • Humidity: Cyclones require a high level of humidity in order to form. This is because the warm, moist air that is drawn into the low-pressure center of the storm helps to fuel its growth and intensity.

 

Question 2

Consider the following statements:

1. The winds which blow between 30 N and 60 S latitudes throughout the year are known as westerlies.

2. The moist air masses that cause winter rains in North-Western region of India are part of  westerlies

Which of the statements given above is/are correct?

(a) 1 only

(b) 2 only

(c) Both 1 and 2

(d) Neither 1 nor 2

Ans: b

Sub-Theme: Wind Movement/Indian Monsoon Option (b) is correct: Generally the westerlies flow between 30° and 60°N and 30° and 60°S

latitude respectively. The moist air masses that

cause winter rains in North-Western region of India are part of westerlies which are also known as winter disturbances.

Westerlies:

  • The westerlies are the winds blowing from the subtropical high pressure belts towards the sub polar low pressure belts.
  • The Westerlies play an important role in carrying the warm, equatorial waters and winds to the western coasts of continents that is the eastern section of the Oceans in the temperate zone.

Note: In this question, the examiner made a simple but difficult to observe change in latitudes i.e “30 N and 60 S latitudes” to make the statement incorrect. Always read questions carefully and cautiously. Although we read incorrect statements in the exam hall, our mind correlates it to what we have read in the textbook/standard book, making it difficult to identify minor twists and twicks. Earlier, UPSC had asked questions on westerlies in 2011. Revising PYQ and doing peripheral research is always important from prelims perspective.

 

Question 3

“Each day is more or less the same, the morning is clear and bright with a sea breeze; as the Sun climbs high in the sky, heat mounts up, dark clouds form, then rain comes with thunder and lightning. But the rain is soon over.”

Which of the following regions is described in the above passage?

(a) Savannah

(b) Equatorial

(c) Monsoon

(d) Mediterranean

Ans: (b)

Sub-Theme: Biomes/World Climate

Option (b) is correct: The above sentence indicates the characteristics of the Equatorial region.

Equatorial Climate:

  • Equatorial climate is typically hot and humid, with little seasonal variation in temperature. These regions are located near the equator, including parts of South America, Africa, and Asia. Precipitation is high and frequent throughout the year, with some areas receiving over 100 inches (250 cm) annually.These regions are also prone to tropical storms and cyclones, especially in coastal areas, which can cause significant damage to infrastructure and
  • Each day is more or less the same, the morning is clear and bright with a sea breeze; as the Sun climbs high in the sky, heat mounts up, dark clouds form, then rain comes with thunder and lightning. But the rain is soon over.

NOTE: Again a direct question came from GC Leong. Also please learn to identify keywords/keyphrases, anyone with a decent reading of NCERTs and GC Leong could easily answer this question. Questions on Equatorial climate have been repeated in 2013, therefore for detailed explanation on Equatorial Biomes please refer to respective PYQs.

 

Question 4

What explains the eastward flow of the equatorial counter current?

(a) The Earth’s rotation on its axis

(b) Convergence of the two equatorial currents

(c) Difference in salinity of water

(d) Occurrence of the belt of calm near the equator

Ans: b

Sub-Theme: Equatorial Counter Currents

Option (a) is incorrect: As the earth rotates from west to east, water near the equator tends to move from east to west (due to the prevailing winds). This leads to the formation of Equatorial currents, both North and South of the equator.

Option (b) is correct: Piling up of water in the western pacific due to the convergence of the North Equatorial Current and South Equatorial Current. This piled up water then flows from west to east, under the effect of gravity, resulting in Counter Equatorial Current (CEC).

Option (c) is incorrect: Difference in salinity of water greatly influences vertical currents and its influence on horizontal movement is less significant.

Option (d) is incorrect: The Doldrums/ Equatorial belt around the Equator are zones of very calm winds, it has little effect on eastward flow of equatorial counter current.

 

Question 5

Tides occur in the oceans and seas due to which among the following?

  1. Gravitational force of the Sun

  2. Gravitational force of the Moon

  3. Centrifugal force of the Earth

Select the correct answer using the code given below.

(a) 1 only

(b) 2 and 3 only

(c) 1 and 3 only

(d) 1, 2 and 3

Ans: d

Sub-Theme: Formation of Tides

56

  • Tides: The “tide” refers to the daily or twice- daily cyclic rise and fall of the sea level, which is primarily caused by the gravitational pull of the moon and the sun. Due to the significant fluctuations in frequency, size, and height of tides, studying them requires a tremendous deal of geographical and temporal complexity.
  • Tides occur due to a balance between all these forces such as Gravitational force of the Sun; Gravitational force of the Moon and Centrifugal force of the Earth.

 

Question 6

In India, the steel production industry requires the import of:

(a) Saltpetre

(b) Rock phosphate

(c) Coking coal

(d) All of the above

Ans: c

Sub-Theme: Industry

Option (a) is incorrect: Saltpetre (Potassium nitrate) is not used in the production of Steel. It is used in the production of fertilizers, tree stump removal, rocket propellants and fireworks.

Option (b) is incorrect: Rock phosphate is used in the production of fertilizer and not for the production of Steel.

Option (c) is correct: The production of steel requires 0.8 tonnes of coking coal to produce one tonne of Steel. And about 85% of the coking coal requirement of the domestic steel industry is presently being met through imports.

 

Question 7

In India, in which one of the following types of forests is teak a dominant tree species?

(a) Tropical moist deciduous forest

(b) Tropical rainforest

(c) Tropical thorn scrub forest

(d) Temperate forest with Grasslands

Ans: a

Sub-Theme: Natural Vegetation of India About Tropical Moist Deciduous Forest:

Climatic Conditions:

  • Annual rainfall 100 to 200 cm.
  • Mean annual temperature of about 27°C
  • The average annual relative humidity of 60 to 75 per cent.
  • Spring (between winter and summer) and summer are dry.

10 1

Characteristics:

    • In the spring and early summer, when there is insufficient rainfall, trees lose their
    • Extreme summers leave everything in a state of bareness (April-May).
    • Tropical moist deciduous forests have an irregular top storey that ranges from 25 to 60 meters.

 

Question 8

Consider the following rivers:

  1. Vamsadhara

  2. Indravati

  3. Pranahita

  4. Pennar

Which of the above are tributaries of Godavari?

(a) 1, 2 and 3

(b) 2, 3 and 4

(c) 1, 2 and 4

(d) 2 and 3 only

Ans: d

Sub-Theme: Peninsular Drainage/River

About River Godavari:

  • The Godavari is India’s second longest and third-biggest river, after the Ganga.
  • Through a vast network of tributaries, the river eventually drains into the Bay of
  • The Godavari is the greatest river in peninsular India in terms of length, catchment area, and flow, and has been nicknamed the Dakshina Ganga, or “Southern ”
  • Indravati and Pranhita are the tributaries of Godavari.

11 1

Additional Information:

  • River Vamsadhara is an important east- flowing river between Rushikulya and Godavari, in Odisha and Andhra Pradesh. It is not a tributary of Godavari.
  • The Pennar rises in the Chennakesava hill of the Nandidurga range, in Chikkaballapura district of Karnataka and flows towards east eventually draining into the Bay of Bengal.

 

Question 9

In a particular region in India, the local people train the roots of living trees into robust bridges across the As the time passes, these bridges become stronger. These unique ‘living root bridges’ are found in

(a) Meghalaya

(b) Himachal Pradesh

(c) Jharkhand

(d) Tamil Nadu

Ans: a

Sub-Theme: Living Root Bridges

Living root bridges also known as Jing Kieng Jri are found in Meghalaya are the aerial bridges that are built by weaving and manipulating the roots of the Indian rubber tree.

12

 

About Living Root Bridges:

  • In Meghalaya, one can find living root bridges. The aerial bridges, also referred to as Jing Kieng Jri, are constructed by tying and shaping the roots of Indian rubber
  • A living root bridge resembling a suspension bridge created by guiding the rubber fig tree’s (Ficus elastica) flexible roots across a stream or river and allowing them to develop and become stronger.

 

Question 10

Consider the following States:

  1. Arunachal Pradesh

  2. Himachal Pradesh

  3. Mizoram

In which of the above States do ‘Tropical Wet Evergreen Forests’ occur?

(a) 1 only

(b) 2 and 3 only

(c) 1 and 3 only

(d) 1, 2 and 3

Ans: c

Sub-Theme: Natural Vegetation of India

Option 1 and 3 are correct: In India, tropical wet evergreen forests are found on the eastern and western slopes of the Western Ghats in such states like Tamil Nadu, Karnataka, Kerala and Maharashtra. And also found in Assam, Arunachal Pradesh, Meghalaya, Nagaland, Tripura, Mizoram, West Bengal and Andaman and Nicobar Islands.

Option 2 is incorrect: Himachal Pradesh comes under the Montane type of forest.

 

Question 11

Which one of the following regions of India has a combination of mangrove forest, evergreen forest and deciduous forest?

(a) North Coastal Andhra Pradesh

(b) South-West Bengal

(c) Southern Saurashtra

(d) Andaman and Nicobar Islands

Ans: d

Sub-Theme: Natural vegetation of India

About Andaman and Nicobar Islands:

The Andaman and Nicobar Islands have a tropical rainforest canopy. The Middle Andamans harbours mostly moist deciduous forests. North

Andamans is characterised by the wet evergreen type, with plenty of woody climbers. Also The coastal regions have mangrove forests.

 

Question 12

Which one of the following pairs of States of India indicates the easternmost and westernmost State?

(a) Assam and Rajasthan

(b) Arunachal Pradesh and Rajasthan

(c) Assam and Gujarat

(d) Arunachal Pradesh and Gujarat

Ans: d

Sub-Theme: Basic Understanding of Indian map

Most Points of India:

  • Guhar Moti is the westernmost point of India. It is a small village located in the Kutch district of Gujarat.
  • The northernmost point of India is located in the eastern Karakoram range in the Himalayas, near Indira Col, Siachen
  • The easternmost point is Kibithu in Arunachal Pradesh.
  • Indira Point in Great Nicobar Island is the southernmost point.

 

Question 13

Consider the following pairs:

Place of Pilgrimage Location
1. Srisailam Nallamala Hills
2. Omakareshwar Satmala Hills
3. Pushkar Mahadeo Hills

Which of the above pairs is/are correctly matched?

(a) 1 only

(b) 2 and 3 only

(c) 1 and 3 only

(d) 1, 2 and 3

Ans: a

Picture9

Sub-Theme: Famous pilgrim places of India

Pair 1 is correct: Srisailam is the shrine of Lord Mallikarjuna on the flat top of Nallamala Hills in Andhra Pradesh.The temple is dedicated to the deities Shiva and Parvati. This temple is one of Lord Shiva’s twelve Jyotirlingas and one of Goddess Parvati’s eighteen Shakti Peethas. The lingam is a representation of Shiva, who is worshiped as Mallikarjuna. Bhramaramba is a representation of Parvati.

Pair 2 is incorrect: Omkareshwar is situated on the Mandhata hills in Madhya Pradesh (ancient name: Shivpuri). The name “Omkareshwar” comes from the way the River Narmada flows here as it circles the Mandhata hill in the shape of a “Om.” Omkareshwar is a Hindu temple where the god Shiva is worshiped. It is one of Shiva’s 12 cherished Jyotirlinga shrines.

Pair 3 is incorrect: The Mahadeo Hills are a range of hills in Madhya Pradesh state of central India. Whereas, Pushkar is situated on the shore of Pushkar Lake in Ajmer, Rajasthan. It is famous for the red spired Brahma Temple.

 

Question 14

Which one of the following countries of South-West Asia does not open out to the Mediterranean Sea?

(a) Syria

(b) Jordan

(c) Lebanon

(d) Israel

Ans: b

Sub-Theme: World Map

Correct answer is Option (b): Jordan is a landlocked country and doesn’t have any opening into the Mediterranean Sea.

13 1

NOTE: In terms of world map, UPSC absolutely loves this region and there are many instances that they have asked questions from this region. Also, This region also very much remains in the news. Therefore, reading newspapers and practicing maps is very important for UPSC.

 

2014

 

Question 1

The seasonal reversal of winds is the typical characteristic of

(a) Equatorial climate

(b) Mediterranean climate

(c) Monsoon climate

(d) All of the above climates

Ans: c

Sub-Theme: Biomes/World Climate The Monsoon Climate (Am):

14

The monsoon climate is a type of weather where it rains a lot.

    • It’s found in India, South-east Africa, Northern Australia, the Guinea Coast of West Africa and the Pacific Coast of Columbia.
    • The seasonal reversal of wind along with alternating intervals of rainfall and drought define the monsoon climate.
    • Due to variances in yearly temperature fluctuations between continents and oceans, the monsoonal circulation system with reversal of wind direction emerges.(c) Sub-Theme: Biomes/World Climate The Monsoon Climate (Am):

 

Question 2

Which of the following phenomena might have influenced the evolution of organisms?

  1. Continental drift

  2. Glacial cycles

Select the correct answer using the code given below.

(a) 1 only

(b) 2 only

(c) Both 1 and 2

(d) Neither 1 nor 2

Ans: c

Sub-Theme: Evolution vs Glacial cycle

Factors that might have influenced the Evolution of Organisms:

  • Continental Drift: The evolution must have been influenced by the movement of some animals due to continental drift, while others must have become stranded in one location. The distinctive primates found in Australia and South America are among the most notable examples of how continental drift affected creature evolution.
  • Glacial Cycles: The evolution of species may have been impacted by the increase in sea level brought on by glacial melt.
  • The final stages of evolution of Genus Homo occurs in the last 3 glacial cycles.

 

Question 3

In India, the problem of soil erosion is associated with which of the following?

  1. Terrace cultivation

  2. Deforestation

  3. Tropical climate

Select the correct answer using the code given below.

(a) 1 and 2 only

(b) 2 only

(c) 1 and 3 only

(d) 1, 2 and 3

Ans: b

Sub-Theme: Soil degradation

  • Soil erosion is the process by which soil is displaced and transported from one location to another by natural forces such as wind or water. It is a natural process that occurs over time, but human activities such as deforestation, overgrazing, and construction can accelerate the process, leading to significant environmental damage.

Statement 1 is incorrect: Terracing is the practice of creating nearly level areas in a hillside area. protected from erosion by other soil barriers.

Statement 2 is correct: Deforestation is one of the major reasons for soil erosion. It includes cutting and felling of trees, removal of forest litter. Browsing and trampling by livestock, forest fires, also leads to deforestation, etc.

Statement 3 is incorrect: In the tropical monsoon climate people are mainly engaged in agriculture. Crops growing: rice, wheat, pulses, cotton, jute, sugarcane, oilseeds, coffee, tea and various types of fruits and vegetables.

 

Question 4

Consider the following pairs:

Region Well-known for the production of
1. Kinnaur Areca nut
2. Mewat Mango
3. Coromandel Soya bean

Which of the above pairs is/are correctly matched?

(a) 1 and 2 only

(b) 3 only

(c) 1, 2 and 3

(d) None

Ans: d

Sub-Theme: Famous crop and places

Pair 1 is incorrectly matched: Kinnaur is mostly famous for apples. The production of Areca nuts is mostly confined to Karnataka, Kerala and Assam.

Pair 2 is incorrectly matched: The main occupation of Mewat is agriculture along with allied and agro-based activities, it is not famous for mango. Major mango-growing states are Andhra Pradesh, Uttar Pradesh, Karnataka,

Pair 3 is incorrectly matched: In the Coromandel Coast, major cultivations are Rice, pulses, sugarcane, cotton, peanuts, etc. Production of soybean in India is dominated by Maharashtra and Madhya Pradesh which contribute 89 percent of the total production.

 

Question 5

Consider the following towns of India:

  1. Bhadrachalam

  2. Chanderi

  3. Kancheepuram

  4. Karnal

Which of the above are famous for the production of traditional sarees/fabric?

(a) 1 and 2 only

(b) 2 and 3 only

(c) 1, 2 and 3

(d) 1, 3 and 4

Ans: b

Sub-Theme: Traditional Saree/fabric

Option 1 and 4 are incorrect: Bhadrachalam and Karnal are not famous for the production of traditional sarees/fabric.

Option 2 and 3 are correct: Chanderi sarees is a traditional sari made in Chanderi, Madhya Pradesh, India. Kanjeevaram is a traditional sari from Kanchipuram, Tamil Nadu, India, that is usually handwoven in mulberry silk and has pure gold or silver zari that renders it a festive quality.

 

Question 6

If you travel through the Himalayas, you are likely to see which of the following plants are naturally growing there?

  1. Oak

  2. Rhododendron

  3. Sandalwood

Select the correct answer using the code given below.

(a) 1 and 2 only

(b) 3 only

(c) 1 and 3 only

(d) 1, 2 and 3

Ans: a

Sub-Theme: Natural Vegetation of India

Statement 1 is correct: Evergreen broad-leaf trees, such as oaks and chestnuts, belong to wet temperate types of forests (1000 and 2000 metres).

Statement 2 is correct: Coniferous trees like Rhododendron, pine, deodar, silver fir, spruce and cedar, are found in the temperate forests (1500 and 3000 metres).

Statement 3 is incorrect: Sandalwoods are found in tropical moist deciduous forests or monsoon forests which are found in western ghats, Deccan plateau, northern plains.

NOTE: This is straight from Class 9 Geography NCERT Chapter 5. This underlines the importance of reading NCERTs both for UPSC Pre and Mains perspective.

 

Question 7

With reference to ‘Changpa’ community of India, consider the following statements:

  1. They live mainly in the State of Uttarakhand

  2. They rear the Pashmina goats that yield fine wool

  3. They are kept in the category of Scheduled tribes.

Which of the statements given above is/are correct ?

(a) 1 only

(b) 2 and 3 only

(c) 3 only

(d) 1, 2 and 3

Ans: b

Sub-Theme: Tribe of India

Statement 1 is incorrect: The Changpa are a semi-nomadic people, they are mainly found in the Changtang, a high plateau that stretches across the cold desert of Ladakh.

Statement 2 is correct: They are raised for ultra- fine cashmere wool, also known as pashmina once woven.

Statement 3 is correct: In 1989, Changpas were declared as the Scheduled Tribes.

 

Question 8

Which of the following have coral reefs?

  1. Andaman and Nicobar Islands

  2. Gulf of Kachchh

  3. Gulf of Mannar

  4. Sunderbans

Select the correct answer using the code given below.

(a) 1, 2 and 3 only

(b) 2 and 4 only

(c) 1 and 3 only

(d) 1, 2, 3 and 4

Ans: a

Sub-Theme: Coral Reef

Coral reefs are underwater ecosystems formed by colonies of tiny animals called coral polyps. These animals secrete a hard, calcium carbonate exoskeleton that provides a foundation for the growth of other corals and a diverse array of marine organisms. Coral reefs are found in tropical and subtropical waters around the world, particularly in the Pacific Ocean, the Indian Ocean, and the Caribbean Sea.

Statement 1, 2 and 3 are correct: Coral reefs in India are found in a lot of areas including the Gulf of Kutch, Gulf of Mannar, Palk Bay, Andaman & Nicobar and Lakshadweep Islands.

Statement 4 is incorrect: Sundarban is the largest deltaic region of the world and encompasses over hundreds of islands (105), with a maze of innumerable rivers, rivulets, and creeks but does not have any coral reef.

Picture11

 

Question 9

Consider the following pairs:

National Highway Cities Connected
1. NH4 Chennai and Hyderabad
2. NH6 Mumbai and Kolkata
3. NH15 Ahmedabad and Jodhpur

Which of the above pairs is/are correctly matched ?

(a) 1 and 2 only

(b) 3 only

(c) 1, 2 and 3

(d) None

Ans: d

Sub-Theme: National Highway/Map based

National Highway Cities Connected Length (km)
NH 4 Mayabunder to Port Blair- Chidiyatapu (Andaman) 230
NH 6 Jorabat (Meghalaya) – Selling (Mizoram) 1873
NH 15 Baihat (Assam) – Wakro (Arunachal Pradesh) 664

 

 

Question 10

Which one of the following pairs of islands is separated from each other by the ‘Ten Degree Channel’?

(a) Andman and Nicobar

(b) Nicobar and Sumatra

(c) Maldives and Lakshadweep

(d) Sumatra and Java

Ans: a       

Sub-Theme: Andaman Map based

The 10 degree channel is a water body that separates Andaman in the North and Nicobar in the South. Both the Andaman Islands and Nicobar Islands together form the Indian Union Territory (UT) of Andaman and Nicobar Islands.

Picture12

Additional Information:

  • Nine Degree Channel separates the island of Minicoy from the main Lakshadweep
  • Eight Degree Channel separates the islands of Minicoy and Maldives.
  • Hudson strait separates the Gulf of Hudson & Atlantic Ocean.
  • Palk Strait separates Palk Bay & the Bay of
  • Duncan Passage separates Rutland Island (part of Great Andaman) and Little

 

 

Question 11

Consider the following pairs:

1. Dampa Tiger Reserve : Mizoram

2. Gumti Wildlife Sanctuary : Sikkim

3. Saramati Peak : Nagaland

Which of the above pairs is/are correctly matched?

(a) 1 only

(b) 2 and 3 only

(c) 1 and 3 only

(d) 1, 2 and 3

Ans: c

Sub-Theme: Major places of States

Statement 1 is correct: The Dampa tiger reserve is located in Mizoram. It is located in the Lushai Hills and spans an area of around 500 km2 at an altitude of 800-1,100 m. It is a component of Project Tiger and was designated a tiger reserve in 1994.

Statement 2 is incorrect: Gumti Wildlife Sanctuary is a Wildlife Sanctuary in Tripura. This sanctuary is the place for many animals like elephants, sambar, buffalo, yapping deer, wild goats and numerous more. The sanctuary has become a haven for reptiles as well.

Statement 3 is correct: Saramati peak rising above the surrounding peaks at the mountainous border of Nagaland state, India. It is one of the ultra-prominent peaks of Southeast Asia. It forms a natural boundary between India and Myanmar.

 

Question 12

Consider the following rivers:

  1. Barak

  2. Lohit

  3. Subansiri

Which of the above flows/flow through Arunachal Pradesh?

(a) 1 only

(b) 2 and 3 only

(c) 1 and 3 only

(d) 1, 2 and 3

Ans: b

Sub-Theme: Brahmaputra and its tributary

Statement 1 is incorrect: Barak river is one of the major rivers of South Assam. The 564 km long river is part of the Surma-Meghna River System. It rises in the Manipur Hills in northern Manipur state, India, where it is called the Barak. The Barak River flows through the states of Manipur, Nagaland, Mizoram, and Assam in India and into the Bay of Bengal.

Statement 2 is correct: Lohit River is a tributary of the Brahmaputra River. It flows into Arunachal Pradesh after starting from eastern Tibet’s Zayal Chu range.

Statement 3 is correct: Both the Tibetan Autonomous Region of China and the Indian states of Assam and Arunachal Pradesh are traversed by the Subansiri River. It is the biggest tributary of River Brahmaputra.

Picture13 1

123

 

Question 13

Consider the following pairs:

Hills Region
1. Cardamom Hills Coromandel Coast
2. Kaimur Hills Konkan Coast
3. Mahadeo Hills Central India
4. Mikir Hills North-East India

Which of the above pairs are correctly matched?

(a) 1 and 2

(b) 2 and 3

(c) 3 and 4

(d) 2 and 4

Ans: c

Sub-Theme: Physiography of India/Map based

Pair 1 is incorrect: Cardamom Hills are located in Kerala and south-west Tamil Nadu. It is situated on India’s southwest coast, whereas, the Coromandel coast is located in Tamil Nadu’s eastern coastal plain

971

Pair 2 is incorrect: Kaimur Hills is in the eastern portion of the Vindhya Range, present in Madhya Pradesh, Uttar Pradesh and Bihar.

972

Pair 3 is correct: Mahadeo Hills are located in the northern part of the Satpura Range in Madhya Pradesh state of central India.

Pair 4 is correct: Mikir Hills can be found in Assam, south of Kaziranga National Park. It is a region of the Karbi- Plateau in northeastern India. The highest mountain in the Mikir Hills is Dambuchko.

 

 

Question 14

Turkey is located between:

(a) Black Sea and Caspian Sea

(b)  Black Sea and Mediterranean Sea

(c) Gulf of Suez and Mediterranean Sea

(d) Gulf of Aqaba and Dead Sea

Ans: b

Sub-Theme: World Map

Option (b) is correct: Turkey is located between Black Sea and Mediterranean Sea.

About Geographic Boundary of Turkey:

Turkey is bordered on its northern side by the Black Sea, on its northeast by Georgia and Armenia, on its east by Azerbaijan and Iran, on its southeast by Iraq and Syria, on its southwest and western edges by the Mediterranean Sea and the Aegean Sea, and on its northwest by Greece and Bulgaria.

Picture17 1

 

Question 15

What is the correct sequence of occurrences of the following cities in South-East Asia as one proceeds from south to north?

  1. Bangkok

  2. Hanoi

  3. Jakarta

  4. Singapore

Select the correct answer using the code given below:

(a) 4-2-1-3

(b) 3-2-4-1

(c) 3-4-1-2

(d) 4-3-2-1

Ans: c

Sub-Theme: Major cities/capital cities

The correct sequence of cities in South-East Asia as one proceeds from south to north would be Jakarta > Singapore > Bangkok > Hanoi.

 

2013

 

Question 1

Variations in the length of daytime and night time from season to season are due to:

(a) The earth’s rotation on its axis

(b) The earth’s revolution around the sun in an elliptical manner

(c) Latitudinal position of the place

(d) Revolution of the earth on a tilted axis

Ans: d

Sub-Theme: Motion of the Earth

Option (d) is correct: Variations in the length of daytime and night time from season to season due to revolution of the earth on a tilted axis. Movement of the earth around the sun in a fixed path or orbit.It takes 365¼ days (one year) to revolve around the sun. As a result of the earth’s revolution, you can observe that there are days and nights as well as seasonal fluctuations.

 

Question 2

Which of the following is/are unique characteristic/ characteristics of equatorial forests?

  1. Presence of tall, closely set trees with crowns forming a continuous canopy
  2. Coexistence of a large number of species
  3. Presence of numerous varieties of epiphytes

Select the correct answer using the codes given below:

(a) 1 only

(b) 2 and 3 only

(c) 1 and 3 only

(d) 1, 2 and 3

Ans: d

Sub-Theme: Major Climate of the World

Statement 1 is correct: The tropical rain forest appears from above as a dense canopy of greenery, only disturbed by huge rivers or areas cleared for farming.

Statement 2 is correct: The higher temperatures in the tropics cause higher rates of metabolism, ecological dynamics and coevolutionary processes, which generate and maintain higher biodiversity, which results in the coexistence of a large number of species.

Statement 3 is correct: All plants (most epiphytes) strive upwards for sunshine, resulting in an odd layer structure. Numerous evergreen trees that produce tropical hardwoods like mahogany, ebony, dyewoods, etc. make up the equatorial vegetation.

Natural Vegetation:

  • High temperature and abundant rainfall support a luxuriant tropical rainforest.
  • In the Amazon lowlands, the forest is so dense that it is called ‘Selvas’.

 

Question 3

The annual range of temperature in the interior of the continents is high as compared to coastal areas. What is/are the reason/reasons? 

  1. Thermal difference between land and water

  2. Variation in altitude between continents and oceans

  3. Presence of strong winds in the interior

  4. Heavy rains in the interior as compared to coasts

Select the correct answer using the codes given below:

(a) 1 only

(b) 1 and 2 only

(c) 2 and 3 only

(d) 1, 2, 3 and 4

Ans: a

Sub-Theme: Distribution of the temperature of Earth

Statement 1 is correct: One major factor affecting the distribution of the temperature of Earth is the distribution of Land and Oceans. The loss of heat from the continents is greater than that from the oceans because there is more land in the northern hemisphere and more water in the southern hemisphere, and because the specific heat of land and water differs greatly.

Statement 2 is not correct: The variation in altitude between continents and oceans is not a valid reason for the high annual range of temperature in the interior of the continent as compared to coastal areas. This is because as compared to oceans, the lands heats up and cools down faster.

Statement 3 is not correct: Annual precipitation is usually low in the interior areas as well as the presence of weak winds in the interior.

Statement 4 is not correct: The interiors in Central Asia receive very poor rainfall due to distance from the sea.

 

Question 5

“Climate is extreme, rainfall is scanty and the people used to be nomadic herders.”

The above statement best describes which of the following regions?

(a) African Savannah

(b) Central Asian Steppe

(c) North American Prairie

(d) Siberian Tundra

Ans: b

Sub-Theme: Biomes/World Climate

Steppe Climate or Temperate Continental Climate or Temperate Grassland Climate:

18

Temperature:

  • Continental climate with extreme
  • Temperatures vary  greatly  between summer and winter.

Precipitation:

  • The average rainfall is about 45 cm, but varies as per location from 25 cm to 75 cm.
  • The heaviest rain in June and July (late spring and early summer).

Nomadic herding in Asian Steppes

  • The migratory animal grazing has almost disappeared from the major grasslands. Earlier the nomadic herders were wandering tribes e.g. the Kirghiz, and the Kazakhs
NOTE: Learn to pick keywords/keyphrases from the sentence. Here – ‘extreme climate’; ‘scanty rainfall’; ‘nomadic herders’, etc. all these keywords resemble the characteristics of the Central Asian Steppe region. Earlier UPSC had asked questions about Tropical Savannah Region and equatorial climate in 2012 and 2013 respectively. So preparing about various Biomes/World Climate and their significant features is crucial.

 

Question 6

During a thunderstorm, the thunder in the skies is produced by the:

  1. Meeting of cumulonimbus clouds in the sky.

  2. Lightning that separates the nimbus cloud.

  3. Violent upward movement of air and water particles.

Select the correct answer using the codes given below:

(a) 1 only

(b) 2 and 3

(c) 1 and 3

(d) None of the above produces the thunder

Ans: d

Sub-Theme: Wind Movement

Statement 1 is not correct: Thunder is the sound caused by a lightning discharge. Lightning heats the air in its path and causes a large over- pressure of the air within its channel. The channel expands supersonically into the surrounding air as a shock wave and creates an acoustic signal that is heard as thunder.

Statement 2 is not correct: The thunderstorms are associated with the cumulonimbus clouds not nimbus.

Statement 3 is not correct: The updraft and downdraft determine the path of the thunderstorm.

Additional Information:

  • Cumulonimbus clouds are tall and dense, and are often associated with thunderstorms. They can extend up to 10 miles high in the atmosphere and are characterized by a large, anvil-shaped top. Cumulonimbus clouds can produce heavy rain, hail, lightning, and tornadoes.
  • On the other hand, Nimbus clouds are low-level clouds that typically produce light to moderate rain or snow. They are often found in areas of stable atmospheric conditions and are not typically associated with thunderstorms. Nimbus clouds can appear as a uniform layer of gray or white clouds covering the sky.

 

Question 7

Which of the following statements regarding laterite soils of India are correct?

  1. They are generally red in colour.

  2. They are rich in nitrogen and potash.

  3. They are well-developed in Rajasthan and UP.

  4. Tapioca and cashew nuts grow well on these Soils.

Select the correct answer using the codes given below:

(a) 1, 2 and 3

(b) 2, 3 and 4

(c) 1 and 4

(d) 2 and 3 only

Ans: c

Sub-Theme: Soil Characteristics

Statement 1 is correct: Laterite soil is reddish- brown in colour due to the presence of iron oxide.

Statement 2 is incorrect: Laterite soil is rich in bauxite or ferric oxide. Poor in lime, magnesia, potash and nitrogen.

Statement 3 is incorrect: The laterite soils are commonly found in Karnataka, Kerala, Tamil Nadu, Madhya Pradesh and the hilly areas of Orissa and Assam.

Statement 4 is correct: Suitable crops grown in laterite soil are Groundnut, cashew nut, coffee, rubber, cinchona, arecanut, tapioca, etc.

 

Question 8

Consider the following statements:

  1. Natural gas occurs in the Gondwana beds.

  2. Mica occurs in abundance in Kodarma.

  3. Dharwars are famous for petroleum.

Which of the statements given above is/are correct?

(a) 1 and 2

(b) 2 only

(c) 2 and 3

(d) None

Ans: b

Sub-Theme: Gondwana System

Statement 1 is incorrect: Gondwana rocks contain nearly 98 per cent of India’s coal reserves.

Statement 2 is correct: Koderma is a well-known place for mica production in Jharkhand.

Statement 3 is incorrect: The Dharwar system is economically the most important rocks because they possess valuable minerals like high-grade iron-ore, manganese, copper etc not petroleum.

Picture18

 

Question 9

Consider the following:

  1. Electromagnetic radiation

  2. Geothermal energy

  3. Gravitational force

  4. Plate movements

  5. Rotation of the earth

  6. Revolution of the earth

Which of the above are responsible for bringing dynamic changes on the surface of the earth?

(a) 1, 2, 3 and 4 only

(b) 1, 3, 5 and 6 only

(c) 2, 4, 5 and 6 only

(d) 1, 2, 3, 4, 5 and 6

Ans: d

Sub-Theme: Dynamic Changes of The Earth

Factors of The Dynamic Changes of The Earth:

  • Electromagnetic Radiation: The radiation travels in waves and has electric and magnetic It consists of X-rays, gamma rays, infrared light, visible light, ultraviolet light, radio waves, and microwaves.
  • Geothermal Energy: The primary driving force behind endogenic geomorphic processes is energy coming from within the ground. The primary sources of this energy are radioactivity, friction caused by tidal and rotational motion, and primordial heat from the earth’s formation.
  • Gravitational Force: In addition to being a directional force that activates all downward motions of matter, gravity stresses the constituent parts of the earth. Eg: Mass movement, Avalanche.
  • Plate Movements: Earthquakes and continent building are the results of plate
  • Rotation of the Earth: The unequal heating of the earth’s surface, which arises from rotation about its axis, creates pressure differences and generates winds.
  • Revolution of the Earth: Earth rotates in an elliptical orbit around the sun. The term “Aphelion” refers to the period when the distance between the sun and Earth is at its greatest and “Perihelion” to the period when it is at its shortest.
  • Thus, all of the above are responsible for bringing dynamic changes on the surface of the earth.

 

Question 10

Contour bunding is a method of soil conservation used in:

(a) Desert margins, liable to strong wind action

(b) Low flat plains, close to stream courses, liable to flooding

(c) Scrublands, liable to spread of weed growth

(d) None of the above

Ans: d

Sub-Theme: Soil Conservation

Option (d) is correct: Contour bunding is a method of soil conservation used in Terrace Farming.

  • Terracing and contour bunding which divide the hill slope into numerous small slopes, check the flow of water, promote absorption of water by soil and save soil from erosion.
  • Soil Conservation: Preventing soil erosion or diminished fertility due to excessive use, acidification, salinization, or other chemical soil contamination is known as soil conservation. The biggest single threat to Indian agriculture and animal husbandry is soil erosion.
  • Contour Bunding: Construction of banks along contours is referred to as contour bunding. In affluent areas, this technique for conserving soil and water is Some other methods of soil conservation:Some other methods like Crop,Strip cropping, No- till farming, Contour ploughing,Terrace farming.

 

Question 11

Which of the following adds nitrogen to the soil?

  1. Excretion of urea by animals

  2. Burning of coal by man

  3. Death of vegetation

Select the correct answer using the codes given below:

(a) 1 only

(b) 2 and 3 only

(c) 1 and 3 only

(d) 1, 2 and 3

Ans: c         

Sub-Theme: Soil Fertility

Statements 1 and 3 are correct: Animal waste like urea, uric acid and death of vegetation add nitrogen in the form of nitrates directly into the soil.

Statement 2 is incorrect: Burning coal releases CO, CO2, sulphur dioxide and oxides of nitrogen – air pollutants to the atmosphere.

 

Question 12

Which of the following is/are the characteristic/ characteristics of Indian coal?

  1. High ash content

  2. Low sulphur content

  3. Low ash fusion temperature

Select the correct answer using the codes given below:

(a) 1 and 2 only

(b) 2 only

(c) 1 and 3 only

(d) 1, 2 and 3

Ans: a

Sub-Theme: Major Mineral

Option (a) is correct: The India Coal is mainly characterised by High Ash COntent and Low Sulphur Content.

  • Coal is the most widely used fossil fuel in India. It meets a sizable portion of the country’s energy requirements.
  • It is used to generate power, provide energy to industries, and meet domestic demands.
  • Indian coals have a high ash level but a low sulphur content by nature.
  • Indian coal has a high ash fusion temperature of about 1,500°C.
  • In India, bituminous coal accounts for over 80% of the non-coking quality coal

 

Question 13

Consider the following crops

  1. Cotton

  2. Groundnut

  3. Rice

  4. Wheat

Which of these are Kharif crops?

(a) 1 and 4 only

(b) 2 and 3 only

(c) 1, 2 and 3 only

(d) 2, 3 and 4 only

Ans: c

Sub-Theme: Major Crops

Option (c) is correct: Cotton, Groundnut and Rice are Kharif Crops in India.

About Kharif Crops:

  • Kharif crops are sown during June-July and harvested during September-October period. Rice, maize, jowar, bajra, moong, tur, urad, jute, cotton, soybean, groundnut, are the type of kharif crops.
  • Rice: Temperature-22-32 degree celsius, Rainfall-1500-300 cm, Soil-Deep clay and loamy soil.
  • Cotton: Temperature 21-30 degree celsius, Rainfall 50-100 cm, Soil-well drained light sandy loams, yellow and black soils.
  • Groundnut: Temperature-30-35 degree celsius, Rainfall-40-125 cm,Well drained sandy loam soil

About Rabi Crops:

  • Rabi crop is sown during October- December month and harvested in April- June. Wheat, barley, peas, gram, mustard etc are types of Rabi crops.
  • Wheat: Temperature 10-15 degree celsius, Rainfall 50-100 cm, Soil-well drained fertile loamy soil.

 

Question 14

Which one among the following industries is the maximum consumer of water in India?

(a) Engineering

(b) Paper and pulp

(c) Textiles

(d) Thermal power

Ans: d

Sub-Theme: Water consumption Consumption of water in various industries:

  • Engineering consumes nearly 2019.9 million cubic metres annually.
  • Paper and  pulp  consume  nearly 905.8 million cubic metres annually.
  • Textiles consume nearly 8 million cubic metres annually.
  • Thermal power plants consume nearly 35,157.4 million cubic metres annually.

 

Question 15

Consider the following pairs:

Tribe States
1. Limboo (Limbu) Sikkim
2. Karbi Himachal Pradesh
3. Dongaria Kondh Odisha
4. Bonda Tamil Nadu

Which of the above pairs are correctly matched?

(a) 1 and 3 only

(b) 2 and 4 only

(c) 1, 3 and 4 only

(d) 1, 2, 3 and 4

Ans: a

Sub-Theme: Tribes of India

Pair 1 is correct: Tamang , Limbu, Bhutia, Khas, Lepchas are the tribal community of Sikkim.

Pair 2 is incorrect: Karbi is a tribal community from Karbi Anglong of Assam.

Pair 3 is correct: Dongaria tribal community is from Odisha.

Pair 4 is incorrect: Bonda is the most primitive tribal group in Odisha.

About Tribal Community and States:

  • Dongaria: Odisha
  • Karbi: Assam
  • Bonda: the most primitive tribal groups in
  • Tamang , Limbu, Bhutia, Khas, Lepchas:

Sikkim

 

Question 16

The Narmada River flows to the west, while most other large peninsular rivers flow to the Why?

  1. It occupies a linear rift valley.

  2. It flows between the Vindhyas and the satpuras.

  3. The land slopes to the west from Central India.

Select the correct answer using the codes given

(a) 1 only

(b) 2 and 3

(c) 1 and 3

(d) None

Ans: a

Sub-Theme: West Flowing Peninsular Rivers

Statement 1 is correct: Narmada and Tapi flow through faults (linear rift, rift valley, trough) created due to the bending of the northern peninsula during the formation process of Himalayas.

Statement 2 is incorrect: Narmada flows through the faults, which run parallel to the Vindhyas and the Satpuras.

Statement 3 is incorrect: Narmada flows towards the west not because of land slopes to the west from central India, but it is flowing west because of the rift valleys only.

 

Question 17

Consider the following pairs:

National Park River flowing

through the Park

1. Corbett         National Park Ganga
2. Kaziranga National Park Manas
3. Silent Valley National Park Kaveri

Which of the above pairs is/are correctly matched?

(a) 1 and 2

(b) 3 only

(c) 1 and 3

(d) None

Ans: d

Sub-Theme: Geography of Protected Areas

Pair 1 is incorrect: Ramganga, Sonanadi, Mandal, Palain and Kosi are the major rivers flowing through the Corbett National Park.

Pair 2 is incorrect: The Kaziranga National Park is circumscribed by the Brahmaputra River, which forms the northern and eastern boundaries, and the Mora Diphlu, which forms the southern boundary. Other notable rivers within the park are the Diphlu and Mora Dhansiri.

Pair 3 is incorrect: The Kunthipuzha River drains the entire 15 km length of the Silent Valley park from north to south into the Bharathapuzha River.

 

Question 18

Consider the following pairs:

  1. Nokrek Biosphere Reserve – Garo Hills

  2. Logtak (Loktak) Lake – Barail Range

  3. Namdapha National Park – Dafla Hills

Which of the above pairs is/are correctly matched?

(a) 1 only

(b) 2 and 3 only

(c) 1, 2 and 3

(d) None

Ans: a

Sub-Theme: Map based

Picture19 1

Pair 1 is correct: Nokrek Biosphere Reserve is located in the Garo Hills district of Meghalaya. The biosphere reserve’s name comes from Nokrek Hill, the highest peak of the Garo Hills. Nokrek National Park was included on UNESCO’s list of biosphere reserves in May 2009.

Pair 2 is incorrect: The Loktak Lake is a freshwater lake in Manipur, it is well-known for the phumdis that float above it. On this lake is Keibul Lamjao National Park, the only floating national park in the world. The northern, middle, and southern zones of the lake are separated.

Pair 3 is incorrect: Namdapha National Park is situated between the Dapha Bum range and the Patkai range in the Changlang district of Arunachal Pradesh. The Noa Dihing River runs east to west through the national park. It has dipterocarp woods, the northernmost lowland evergreen rainforests in the world.

 

Question 19

Which one of the following pairs is correctly matched?

Geographical Feature Region
(a) Abyssinian Plateau Arabia Arabia
(b) Atlas Mountains North-Western Africa
(c) Guiana Highlands South-Western Africa
(d) Okavango Basin Patagonia

Ans: b

Sub-Theme: World Map

Atlas Mountain: The Maghreb, often referred to as Northwest Africa, the Greater Maghreb, and historically as the Barbary Coast, is where the Atlas Mountains is situated. The Atlas Mountains divide the Sahara Desert from the Mediterranean and Atlantic coasts. The highest point of this mountain range, Toubkal (4167 m), is found in Morocco. The mountains are found in Algeria and Tunisia in addition to Morocco. The Atlas Mountains are home to populations of Berber people.

Additional Information:

Picture20

Abyssinian Plateau: The Ethiopian Plateau is also referred to as Altipiano Etiopico, Amhara Plateau, Ethiopian Massif, and Ethiopian Plateau. The Abyssinian Plateau was formerly known as Ethiopia, hence the name. The Blue Nile, which emerges from Lake Tana, is the most notable river to cross the plateau.

Picture21

Okavango Basin: The Okavango Basin is situated in southwest Africa comprising Botswana, Namibia, Zimbabwe, Angola, and other countries. Whilst, Patagonia refers to a geographical region that encompasses the southern end of South America, governed by Argentina and Chile.

Guiana Highlands: In South America, the Guiana Highlands are situated in the south of the Orinoco River and north of the Amazon River. Parts of Venezuela, the Guianas, Brazil, and Colombia are included in the Guiana Highlands. Mount Roraima, located in the Guiana Highlands, has the highest elevation (2772 m).

Picture22

 

2012

 

Question 1

A person stood alone in a desert on a dark night and wanted to reach his village which was situated 5 km east of the point where he was standing. He had no instruments to find the direction but he located the polestar. The most convenient way now to reach his village is to walk in the

(a) Direction facing the polestar

(b) Direction opposite to the polestar

(c) Direction keeping the polestar to his left

(d) Direction keeping the polestar to his right

Ans: c

Sub-Theme: Basics of Solar System

Option (a) is incorrect: The North star/Pole Star indicates the north direction, therefore walking in the direction of the pole star will lead him towards NORTH of his current position.

Option (b) is incorrect: Walking in the opposite direction will lead him towards SOUTH of his current position.

Option (c) is correct: Walking in the direction while keeping the polestar to his left would lead him towards EAST of his current position and he can reach his village.

Option (d) is incorrect: Walking in the direction while keeping the pole star to his right would lead him towards WEST of his current position.

1 5

Note: The Pole star is not visible from the southern hemisphere. Some of the north- ern constellations like Ursa Major may also not be visible from some points in the south- ern hemisphere.

 

Question 2

Electrically charged particles from space traveling at speeds of several hundred km/sec can severely harm living beings if they reach the surface of the Earth. What prevents them from reaching the surface of the Earth?

(a) The Earth’s magnetic field diverts them towards its poles.

(b) Ozone layer around the Earth reflects them back to outer space.

(c) Moisture in the upper layers of the atmosphere prevents them from reaching the surface of the

(d) None of the statements (a), (b), and (c) given above is correct.

Ans: a

Sub-Theme: Basics of Solar System

Earth’s Magnetic Field Diverts Electrically Charged Particles:

Option (a) is correct: Earth’s field lines start near the South Pole of the Earth, curve around in space, and converge again near the North Pole, it forms the magnetosphere, which deflects the Sun’s ions and electrons before they reach us. Most of the solar wind is diverted to poles.

Option (b) is incorrect: The ozone layer absorbs a range of ultraviolet energy, it has nothing to do with Electrically charged particles.

Option (c) is incorrect: Moisture in the upper layers of the atmosphere i.e. The Stratosphere prevents Ultraviolet rays from reaching the surface of the earth.

 

Question 3

Normally, the temperature decreases with the increase in height from the Earth’s surface, because

1. The atmosphere can be heated upwards only from the Earth’s surface

2. There is more moisture in the upper atmosphere.

3. The air is less dense in the upper atmosphere.

Select the correct answer using the codes given below:

(a) 1 only

(b) 2 and 3 only

(c) 1 and 3 only

(d) 1, 2 and 3

Ans: c

Sub-Theme: Atmosphere

Statement 1 is correct: Normal lapse rate that is 1-degree Celcius temperature falls at a height of every 165 mt in the troposphere. As we increase elevation, the air pressure decreases as there is less air above us. As the pressure decreases, air molecules spread out further i.e. air expands, and the temperature decreases and the atmosphere can be heated upwards only from the Earth’s surface. Temperature falls off with height at a predictable rate because the air near the surface is heated and becomes light, and the air higher up cools to space and becomes heavy.

Statement 2 is not correct: The water vapour, which accounts for around 1% of air varies greatly in the troposphere and decreases rapidly with altitude.

Statement 3 is correct: The air density in the atmosphere decreases with height and it helps in decreasing temperature.

 

Question 4

Which one of the following is the characteristic climate of the Tropical Savannah Region?

(a) Rainfall throughout the year

(b) Rainfall in winter only

(c) An extremely short dry season

(d) A definite dry and wet season

Ans: d

Sub-Theme: Biomes/World Climate

Option (d) is correct: The Tropical Savannah Region is characterized by a definite dry and wet season.

Tropical Savannah Region:

  • The savanna biome, which is a type of grassland biome, consists of areas of open grassland with very few trees.
  • In the Wet Season, the weather is warm and a savanna receives as much as 50 inches of rain.
  • During the Dry Season, weather can be extremely hot, and rainfall will amount to only four inches each month.
  • This combination of high temperatures and little precipitation makes savannas perfect areas for grass and brush fires during their dry seasons.

 

Question 5

The acidification of oceans is increasing. Why is this phenomenon a cause of concern

1. The growth and survival of calcareous phytoplankton will be adversely affected.

2. The growth and survival of coral reefs will be adversely affected.

3. The survival of some animals that have phytoplanktonic larvae will be adversely

4. The cloud seeding and formation of clouds will be adversely affected.

Which of the statements given above is/are correct?

(a) 1, 2 and 3 only

(b) 2 only

(c) 1 and 3 only

(d) 1, 2, 3 and 4

Ans: a

Sub-Theme: Ocean Acidification

Statement 1 is correct: Ocean acidification ecreases the calcifying ability of corals, calcareous plankton, crustaceans etc.

Statement 2 is correct: Ocean acidification will affect corals and this will, in turn, affect one million species that have made corals their homes.

Statement 3 is correct: Coral reefs will erode faster than they can rebuild. When shelled organisms are at risk, the entire food web may also be at risk.

Statement 4 is incorrect: Atmospheric sulfur is produced and emitted by phytoplankton from the ocean in the form of dimethyl sulfide, the increase in ocean acidification led to an adverse impact on phytoplankton development and which led to a low amount of sulfur emission. This will decrease cloud formation in small amounts and not adversely affect indirectly on cloud formation.

2 3

 

Question 6

Consider the following factors

  1. Rotation of the Earth

  2. Air pressure and wind

  3. Density of ocean water

  4. Revolution of the Earth

Which of the above factors influence the ocean currents?

(a) 1 and 2 only

(b) 1, 2 and 3

(c) 1 and 4

(d) 2, 3 and 4

Ans: b

Sub-Theme: Factors Influence the Ocean Currents

Statement 1 is correct: The Coriolis force is the result of the earth’s rotation. It intervenes and causes the water to move to the right in the northern hemisphere and to the left in the southern hemisphere.

Statement 2 is correct: Wind blowing on the surface of the ocean pushes the surface water along with it and the friction between the wind and the water surface affects the movement of the water body in its course.

Statement 3 is correct: Differences in water density affects vertical mobility of ocean currents. Water with high salinity is denser than water with low salinity and in the same way cold water is denser than warm water. Denser water tends to sink, while relatively lighter water tends to rise.

Statement 4 is incorrect: The revolution of the earth does not influence the ocean currents.

 

Question 7

Which of the following is the chief characteristic of ‘mixed farming’?

(a) Cultivation of both cash crops and food

(b) Cultivation of two or more crops in the same

(c) Rearing of animals and cultivation of crops

(d) None of the above.

Ans: c

Sub-Theme: Agricultural pattern/type

Option (c) is correct: Mixed Farming is a situation in which both raising crops and rearing animals are carried on simultaneously.

Option (a) and (b) are incorrect: Cultivation of both cash crops and food crops and Cultivation of two or more crops in the same field are related to the practice of mixed cropping or inter cropping.

 

Question 8

Consider the following crops of India:

  1. Groundnut

  2. Sesamum

  3. Pearl millet

Which of the above is/are predominantly rainfed crop/crops?

(a) 1 and 2 only

(b) 2 and 3 only

(c) 3 only

(d) 1, 2 and 3

Ans: d

Option (d) is correct: The crops which are predominantly rainfed in India are Groundnut, Sesamum and Pearl millets.

  • Groundnut is a drought-tolerant crop that requires moderate rainfall for its growth and development. Rainfall for cultivation – 500 to 600 mm per annum, distributed rainfall pattern throughout the growing season. Groundnut can withstand a dry spell of 15-20 days during its vegetative stage, but it requires adequate moisture during the flowering and pod formation stages.
  • Sesamum requires temperature is 25°C to 30°C, can tolerate up to 40°C, sensitive to frost required rainfall is 500-600 mm, its drought-tolerant, excessive rainfall during flowering can cause lower yields . It requires well-drained Soil type, rich in organic matter, pH range of 5 to 8.0.
  • Pearl millet is a hardy crop that can grow in semi-arid regions. The ideal rainfall for pearl millet cultivation is between 400 to 600 mm per annum, with a well- distributed rainfall pattern throughout the growing season. Pearl millet can tolerate drought conditions and requires moisture during the early growth stages, especially during germination and tillering.
  • Rainfed areas receive more than 750 mm. Rainfed agriculture is carried out in a wide range of soil types, agro-climatic conditions, and annual rainfall ranges from 400 mm to 1600 mm. Rain-fed areas account for percent of millet production, 88 percent of pulses, 73 percent of cotton, 69 percent of oilseeds including groundnut and Sesame.

NOTE: Question on crops earlier came in CSE-2011. Reading about important crops and their characteristics from NCERT (Class 12th – India people and Economy) is important.

 

Question 9

When you travel in Himalayas, you will see the following:

  1. Deep gorges

  2. U-turn river courses

  3. Parallel mountain ranges

  4. Steep gradients causing land-

Which of the above can be said to be the evidence for Himalayas being young fold mountains?

(a) 1 and 2 only

(b) 1, 2 and 4 only

(c) 3 and 4 only

(d) 1, 2, 3 and 4

Ans: d

Sub-Theme: Physiography of India

Deep Gorges, U-turn river courses, Parallel mountain ranges and steep gradients causing landslides – all these are representations or characterics of Himalayan youthful topography.

About Himalayan Physiography:

  • The Himalayas along with other Peninsular mountains are young, weak and flexible in their geological structure unlike the rigid and stable Peninsular Block.
  • Consequently, they are still subjected to the interplay of exogenic and endogenic forces, resulting in the development of faults, folds and thrust plains.
  • These mountains are tectonic in origin, dissected by fast-flowing rivers which are in their youthful stage.
  • In fact the whole mountain system of Himalaya represents a very youthful topography with high peaks, deep V-shaped valleys, gorges, rapids, waterfalls, steep slopes, etc.

3 3

 

Question 10

Consider the following statements:

  1. The duration of the monsoon decreases from southern India to northern India.

  2. The amount of annual rainfall in the northern plains of India decreases from east to west.

Which of the statements given above is/are correct?

(a) 1 only

(b) 2 only

(c) Both 1 and 2

(d) Neither 1 nor 2

Ans: c

Sub-Theme: Indian Monsoon

  • Monsoon refers to a seasonal change in wind patterns that results in a shift in precipitation patterns over a region. Monsoons occur primarily in tropical and subtropical regions and are characterized by a wet season and a dry season.
  • During the wet season, the winds bring in warm, moist air from over the oceans, which leads to increased rainfall over the land.
  • The dry season, on the other hand, is characterized by a shift in wind patterns, which brings in cooler, drier air from over the land, leading to a decrease in rainfall.

Statement 1 is correct: Due to its proximity to the sea, India’s southern region has earlier and longer rainy seasons than its northern region. Rainfall in the northern section of India diminishes as rain-bearing winds cross the western ghats and humidity decreases. The southwest monsoon and the retreating monsoon (northeast monsoon) bring rain to India’s southern region.

Statement 2 is correct: With growing distance from the sea, monsoon rainfall tends to decrease. During the southwest monsoon, Kolkata receives 119 cm, Patna 105 cm, Allahabad 76 cm, and

Delhi 56 cm.

 

Question 11

A particular State in India has the following characteristics:

1. It is located on the same latitude which passes through northern Rajasthan.

2. It has over 80% of its area under forest

3. Over 12% of forest cover constitutes the Protected Area Network in this State.

Which one among the following States has all the above characteristics?

(a) Arunachal Pradesh

(b) Assam

(c) Himachal Pradesh

(d) Uttarakhand

Ans: a

Sub-Theme: State of India/Map based

  • Arunachal Pradesh has a forest cover of 80% with total areas of 67410 sq km. Over 12% of forest cover constitutes Protected Area Network in Arunachal Pradesh. (Forest Report)
  • Arunachal Pradesh is located on the same latitude which passes through northern
  • Arunachal Pradesh, Assam, Sikkim, Uttar Pradesh, Haryana are the states that lie in the same latitude that passes through

 

Question 12

With reference to the wetlands of India, consider the following statements:

  1. The country’s total geographical area under the category of wetlands is recorded more in Gujarat as compared to other States.

  2. In India, the total geographical area of coastal wetlands is larger than that of inland wetlands.

Which of the statements given above is/are correct?

(a) 1 only

(b) 2 only

(c) Both 1 and 2

(d) Neither 1 nor 2

Ans: a

Sub-Theme: State of India/Wetland

Wetlands:

  • Wetlands are areas where water is the primary factor controlling the environment and the associated plant and animal life. They occur where the water table is at or near the surface of the land, or where the land is covered by water.
  • Under the text of the Ramsar Convention (Article 1), wetlands are defined as: “areas of marsh, fen, peatland or water, whether natural or artificial, permanent or temporary, with water that is static or flowing, fresh, brackish or salt, including areas of marine water the depth of which at low tide does not exceed six metres”.

Statement 1 is correct: The total wetland area in Gujarat is 23.14% of the total wetland of the country.

Statement 2 is incorrect: The area under inland wetlands accounts for 69%, coastal wetlands 27%, and other wetlands (smaller than 2.25 ha) 4%.

NOTE: Please refer to PYQ 2011 for detailed information on the state of Gujarat.

 

2011

 

Question 1

Westerlies in the southern hemisphere are stronger and persistent than in northern hemisphere. Why?

1. Southern hemisphere has less landmass as compared to northern hemisphere.

2. Coriolis force is higher in southern hemisphere as compared to northern hemisphere.

Which of the statements given above is/are correct?

(a) 1 only

(b) 2 only

(c) Both 1 and 2

(d) Neither 1 nor 2

Ans: a

Sub-Theme: Wind System

Option (a) is correct: The Southern hemisphere has less landmass as compared to the northern hemisphere. The Coriolis force is the same in both hemispheres.

Landmass in both the Hemispheres:

  • The Northern Hemisphere’s surface is about 40% landmass and is covered by about 60% water in the Northern
  • The Southern Hemisphere’s surface is about 20% landmass and is covered by 80% water.
  • Circulating air is deflected to the right in the Northern Hemisphere and to the left in the Southern Hemisphere as a result of the Earth’s axis of This is known as the Coriolis effect.

Picture1 8

The latitude and airspeed have a direct correlation with the amount of deflection the air makes. At the equator, the Coriolis force is zero, while maximum at the poles.

 

Question 2

What could be the main reason/reasons for the formation of the African and Eurasian desert belt?

1. It is located in the sub-tropical high-pressure cells.

2. It is under the influence of warm ocean currents.

Which of the statements given above is/are correct in this context?

(a) 1 only

(b) 2 only

(c) Both 1 and 2

(d) Neither 1 nor 2

Ans: a

Sub-Theme: Desert

Option (a) is correct: The main reason/reasons for the formation of the African and Eurasian desert belt is because they are located in the sub-tropical high-pressure cells.

Deserts:

  • Deserts are regions where evaporation exceeds precipitation.
  • The main causes of the desert’s aridity are insufficient and irregular rainfall, high temperatures, and a quick rate of
  • The hot deserts are found on the western coasts of continents, between 15° and 30°N. and S, in the trade wind belt.
  • Offshore trade winds are often bathed in cold currents which produce a desiccating (dehydrating) effect, hence moisture is not easily condensed into precipitation.

2 4 1

 

Question 3

The jet aircraft fly very easily and smoothly in the lower stratosphere. What could be the appropriate explanation?

1. There are no clouds or water vapour in the lower stratosphere.

2. There are no vertical winds in the lower Stratosphere.

Which of the statements given above is/are correct in this context?

(a) 1 only

(b) 2 only

(c) Both 1 and 2

(d) Neither 1 nor 2

Ans: c

Sub-Theme: Layers of Atmosphere

Option (c) is correct: The conditions are practically perfect for flying airplanes since this layer is almost completely devoid of clouds and related weather disturbances. So airplanes fly in the lower stratosphere, sometimes in the upper troposphere where weather is calm.

  • The increase in the temperature with height in the stratosphere makes this region a stable place where the air tends not to overturn vertically. Thus vertical winds are almost absent in Stratosphere

Picture2 4

About Stratosphere:

  • It is located above the troposphere up to 50 km above the surface of the earth.
  • The thickness of the stratosphere is highest at the poles.

 

Question 4

The 2004 Tsunami made people realize that mangroves can serve as a reliable safety hedge against coastal calamities. How do mangroves function as a safety hedge?

(a) The mangrove swamps separate the human settlements from the sea by a wide zone in which people neither live nor venture out

(b) The mangroves provide both food and medicines which people are in need of after any natural disaster.

(c) The mangrove trees are tall with dense canopies and serve as an excellent shelter during a cyclone or tsunami

(d) The mangrove trees do not get uprooted by storms and tides because of their extensive roots.

Ans: d

Sub-Theme: Mangroves

Option (d) is correct: because mangroves function as a safety hedge in preserving ecological stability. The dense tangle of roots allows the trees to withstand storms, cyclones or even the daily rise and fall of tides.

Mangroves:

  • A mangrove is a small tree or shrub that grows along coasts, setting roots in salty sediments, usually beneath water.
  • Mangrove trees can withstand the tides’ daily rise and fall due to their complex web of roots and extensive roots.

 

Question 5

La Nina is suspected to have caused recent floods in How is La Nina different from El Nino?

1. La Nina is characterized by unusually cold ocean temperature in the equatorial Indian Ocean whereas El Niño is characterized by unusually warm ocean temperature in the equatorial Pacific Ocean.

2. El Nino has adverse effects on the south-west monsoon of India, but La Niña has no effect on monsoon climate.

Which of the statements given above is/are correct?

(a) 1 only

(b) 2 only

(c) Both 1 and 2

(d) Neither 1 nor 2

Ans: d

Sub-Theme: El niño & La-nina

Statement 1 is incorrect: La Nina is characterized by unusually cold ocean temperatures in the Equatorial Pacific, compared to El Nino, which is characterized by unusually warm ocean temperatures in the Equatorial Pacific.

Statement 2 is incorrect: La Nina causes drought in Peru and Ecuador, heavy floods in Australia, high temperatures in Western Pacific, Indian Ocean, off the Somalian coast, and good monsoon rains in India. El Nino: It occurs more frequently than La Nina. This event is not an usual cycle, they are not predictable and occur irregularly at 2 to 7 year intervals. When the coastal waters become warmer in the eastern tropical Pacific (El Niño) it leads to the decrease of atmospheric pressure above the oceans.

Picture3 3

 

Question 6

The lower Gangetic plain is characterized by a humid climate with high temperature throughout the year. Which one among the following pairs of crops is most suitable for this region?

(a) Paddy and cotton

(b) Wheat and Jute

(c) Paddy and Jute

(d) Wheat and cotton

Ans: (c)

Sub-Theme: Major crops in India

Option (c) is correct: The crop suitable to be grown in the lower Gangetic plain, which is characterized by a humid climate with high temperature throughout the year, is Paddy and Jute.

  • Paddy: It grows well in the plains of north and north-eastern India, coastal areas and the deltaic regions, where there is an abundance of alluvial clayey soil. A high temperature (above 25°C) and significant humidity are necessary for this kharif crop.
  • Jute: Its cultivation is mainly concentrated in eastern India because of the rich alluvial soil of Ganga-Brahmaputra Jute crops need humid conditions with temperature swings between 24 and 38 degrees Celsius.

 

Question 7

Among the following States, which one has the most suitable climatic conditions for the cultivation of a large variety of orchids with minimum cost of production, and can develop an export oriented industry in this field?

(a) Andhra Pradesh

(b) Arunachal Pradesh

(c) Madhya Pradesh

(d) Uttar Pradesh

Ans: b         

Sub-Theme: State of India

About Orchid in Arunachal Pradesh:

The ecological condition and climate variation in Arunachal Pradesh are so favorable that the state holds about 60% of the Indian variety of orchids. Arunachal Pradesh is often called the Orchid State of India.

 

Question 8

The Brahmaputra, Irrawaddy and Mekong rivers originate in Tibet and flow through narrow and parallel mountain ranges in their upper Of these rivers, Brahmaputra makes a “U’’ turn in its course to flow into India. This “U” turn is due to

(a) Uplift of folded Himalayan series

(b) Syntaxial bending of geologically young Himalayas

(c) Geo-Tectonic disturbance in the tertiary folded mountain chains

(d) Both (a) and (b)

Ans: b        

Sub-Theme: Himalayan River System/Drainage

Brahmaputra makes a “U” turn in its course to flow into India due to the Syntaxial bending of geologically young Himalayas.

About Syntaxial Bends/Syntaxis:

  • The Himalayan mountains’ gently arching ranges on their western and eastern extremities are abruptly bent southward in deep knee bends known as syntaxial
  • The massive mountains seem to bend around a pivotal point on both the ends.
  • The westernmost point is located where the Hindu Kush joins the Karakoram south of the Pamir.

Picture4 3

  • Arunachal Pradesh’s eastern border features a similarly abrupt, almost hairpin curve when the mountain’s strike abruptly shifts from an easterly to a southerly

 

Question 9

A state in India has the following characteristics:

1. Its northern part is arid and semi-arid.

2. Its central part produces cotton.

3. Cultivation of cash crops is predominant over food crops.

Which one of the following states has all of the above characteristics?

(a) Andhra Pradesh

(b) Gujarat

(c) Karnataka

(d) Tamil Nadu

Ans: b

Sub-Theme: State of India

The state of Gujarat is characterised by its northern part having arid and semi arid climate. Its central part produces cotton and the Cultivation of cash crops is predominant over food crops.

About Gujarat (Gandhinagar):

  • Physiography: Dry Desert, Coastal region, Semi-arid Kachchh region towards west,Kathiawar Peninsula (Saurashtra)
  • Major crops: Cotton (30% of total production in the country), Groundnuts, Tobacco-All cash crops.

 

Question 10

Between India and East Asia, the navigation-time and distance can be greatly reduced by which of the following?

1. Deepening the Malacca straits between Malaysia and Indonesia.

2. Opening a new canal across the Kra isthmus between the Gulf of Siam and Andaman Sea.

Which of the statements given above is/are correct?

(a) 1 only

(b) 2 only

(c) Both 1 and 2

(d) Neither 1 nor 2

Ans: b        

Sub-Theme: Map based

Statement 1 is incorrect: The Malacca Strait is situated between Malaysia and Indonesia and is one of the main shipping lanes. The narrow waterway creates a chokepoint between the Indian Ocean and the Pacific Ocean. Hence, the broadening (not Deepening) the Malacca straits between Malaysia and Indonesia will reduce the navigation time and distance.

Statement 2 is correct: Kra isthmus is the narrowest part of the Malay Peninsula. It is bordered to the west by the Andaman Sea and to the east by the Gulf of Thailand. Thus, opening a new canal across the Kra isthmus between the Gulf of Siam and the Andaman Sea can reduce the navigation time and distance.

 

Question 11

Two important rivers- one with its source in Jharkhand (and known by a different name in Odisha), and another, with its source in Odisha- merge at a place only a short distance from the coast of Bay of Bengal before flowing into the This is an important site of wildlife and biodiversity and a protected area.

Which one of the following could be this?

(a) Bhitarkanika

(b) Chandipur-on-sea

(c) Gopalpur-on-sea

(d) Simlipal

Ans: a         

Sub-Theme: Physiography of India

About Bhitarkanika National Park

Picture5 2

  • The Bhitarkanika Mangroves are a mangrove wetland in India’s Orissa state. The Bhitarkanika Mangroves cover an area of 650 km square in the river delta of the Brahmani and Baitarani rivers.
  • It is a Ramsar site and home to the second- largest mangrove forest in India. In the year 1988, it was designated as Bhitarkanika National Park.

Picture6

  • The estuary of the Brahmani, Baitarani, Dhamra, and Mahanadi river systems is where Bhitarkanika is situated. It is situated in Odisha’s Kendrapara district.
  • It is renowned for its mangroves, migratory birds, turtles, estuarine crocodiles, and numerous creeks. It is one of Odisha’s finest biodiversity hotspots.
  • It is said to be home to 70% of the nation’s estuary or saltwater crocodiles, whose conservation efforts began in 1975.
  • Protected Areas: Bhitarkanika National Park; Bhitarkanika Wildlife Sanctuary and Gahirmatha Marine Sanctuary.

 Final Result – CIVIL SERVICES EXAMINATION, 2023.   Udaan-Prelims Wallah ( Static ) booklets 2024 released both in english and hindi : Download from Here!     Download UPSC Mains 2023 Question Papers PDF  Free Initiative links -1) Download Prahaar 3.0 for Mains Current Affairs PDF both in English and Hindi 2) Daily Main Answer Writing  , 3) Daily Current Affairs , Editorial Analysis and quiz ,  4) PDF Downloads  UPSC Prelims 2023 Trend Analysis cut-off and answer key

THE MOST
LEARNING PLATFORM

Learn From India's Best Faculty

      

 Final Result – CIVIL SERVICES EXAMINATION, 2023.   Udaan-Prelims Wallah ( Static ) booklets 2024 released both in english and hindi : Download from Here!     Download UPSC Mains 2023 Question Papers PDF  Free Initiative links -1) Download Prahaar 3.0 for Mains Current Affairs PDF both in English and Hindi 2) Daily Main Answer Writing  , 3) Daily Current Affairs , Editorial Analysis and quiz ,  4) PDF Downloads  UPSC Prelims 2023 Trend Analysis cut-off and answer key

Quick Revise Now !
AVAILABLE FOR DOWNLOAD SOON
UDAAN PRELIMS WALLAH
Comprehensive coverage with a concise format
Integration of PYQ within the booklet
Designed as per recent trends of Prelims questions
हिंदी में भी उपलब्ध
Quick Revise Now !
UDAAN PRELIMS WALLAH
Comprehensive coverage with a concise format
Integration of PYQ within the booklet
Designed as per recent trends of Prelims questions
हिंदी में भी उपलब्ध

<div class="new-fform">







    </div>

    Subscribe our Newsletter
    Sign up now for our exclusive newsletter and be the first to know about our latest Initiatives, Quality Content, and much more.
    *Promise! We won't spam you.
    Yes! I want to Subscribe.